MBE KAPLAN--TORTS Flashcards
skip
skip
skip
skip
skip
skip
- A woman owned a beautiful Siamese cat. The cat had a value of $500. The woman allowed the cat to roam loose in the neighborhood. The cat frequently entered the backyard of a neighbor, who lived in the house next to the woman. The neighbor hated cats. One day, the neighbor looked out his kitchen window and saw the cat chewing on his prized rosebushes.
The neighbor telephoned the woman and told her to retrieve her cat or he would kill it. The woman immediately ran over to the neighbor’s property to get the cat. In the interim, the neighbor loaded his rifle and went outside. The woman saw the neighbor with the gun and said, “Please don’t shoot my cat.” The neighbor responded, “I’m sick and tired of her destroying my roses.” The neighbor then pointed the rifle at the cat. Out of instinct, the woman dived toward the rosebushes to save her beloved cat. At that same moment, the neighbor’s hand shook, and the rifle accidentally went off. The bullet narrowly missed the woman’s head but struck the cat. Seeing her cat being shot, the woman was overcome with grief and became very emotionally distraught, which caused her to suffer a heart attack. Fortunately, the cat survived the shooting but suffered a gunshot wound to her leg.
The woman brings suit to recover damages against the neighbor. Which of the following causes of action would afford the woman her maximum recovery?
(A) Battery.
(B) Conversion.
(C) Trespass to chattels.
(D) Negligent infliction of emotional distress.
- (D) The key to this question is carefully reading the interrogatory or “stem,” which asks you to determine the cause of action that would afford the woman her maximum recovery. The woman’s cat is worth only $500, so that would be the maximum amount she could recover with a conversion or trespass to chattels claim. Choice (D), therefore, is the correct answer here, because a negligent infliction of emotional distress claim would give the woman a chance to recover for her emotional distress and the damages might be much higher than $500. Choice (A) is incorrect because battery requires harmful or offensive contact to a person. Here, the woman’s cat suffered a harmful contact, but the woman herself did not. Choice (B) is incorrect because the damages awarded for a conversion claim would be $500, because that was the value of the chattel. Choice (C) is incorrect because the damages for a trespass to chattels claim wouLd be measured by the diminished value of the chattel, which could not exceed $500 here. Exam Tip: The distinction between conversion and trespass to chattels (especially with regard to damages) is often tested on the Multistate exam.
- A patient underwent heart bypass surgery at a hospital. Members of the patient’s family had donated blood to be used during the operation. The patient told the doctor she would agree to the operation only if any blood given to her during the operation came from the blood donated by her family. When the doctor performed the surgery, he requisitioned blood from the hospital’s blood bank for the patient’s transfusion instead of using her family’s donated blood. It was the customary practice of doctors in the community to use blood from the hospital’s blood bank during surgery. ft was later determined that the hospital blood given to the patient was tainted with HIV. The hospital had not properly screened the blood for the presence of the HIV virus. The patient did not contract the HIV virus.
If the patient asserts an action against the doctor for battery, she will
(A) prevail, because consent to the operation was based upon the use of her family’s blood.
(B) prevail, because the hospital failed to properly screen the blood to detect its contamination.
(C) not prevail, because the patient did not contract the HIV virus.
(D) not prevail, because it was the customary practice of doctors in the community to use blood from the hospital’s blood bank during surgery.
- (A) A physician must obtain the patient’s consent to provide treatment. In some emergency circumstances, the patient’s consent may be implied, such as where the patient is unconscious and needs immediate treatment. Where a patient’s consent is limited in some manner, however, the physician must respectthe patient’s wishes and will be liable for battery if the scope of the patient’s consent is exceeded. Here, choice (A) is correct because the patient’s consent to the operation was based on the use of family blood, so the doctor did not have actual or implied consent to use blood from the hospital bank. Choice (B) is incorrect because it addresses whether the patient could prevail on a negLigence cause of action against the hospital, but this question is about liability for battery rather than negligence. Choice (C) is incorrect because she could hold the doctor liable for battery, even if she did not become HIV-positive. She would be entitled to damages for the unwanted, offensive contact that occurred when she received the transfusion of hospital blood, regardless of her HIV status. Choice CD) is incorrect because the fact that doctors generally used blood from the blood bank would not eliminate the doctor’s obligation to obtain his patient’s consent. The customary practices of doctors in the community would be very relevant if she were suing the doctor for negligence, but this question is about a battery claim, not negligence.
- A man decided to stop at a drive-through hamburger stand for a late snack. As he drove up to the drive- through line, the manager of the hamburger stand informed him through the intercom system that the restaurant was closing and no further orders would be accepted. She told the man that the last car to be served was the one directly in front of him. The man became angry and yelled into the intercom machine, “Listen, babe, I am hungry. I want two cheeseburgers, a large order of fries, and a Coke.” The manager retorted, “I’m terribly sorry, but we cannot accept your order.”
Shortly thereafter, the manager handed the food order to the passengers in the car immediately in front of the man’s. When the man saw the manager serving that car, he became very angry, drove his automobile up to the service window and shouted at the manager, “You can’t do this to me.” When the manager laughed, the man suddenly reached into the car’s glove compartment and pulled out a gun. He aimed at the manager and fired the weapon, intending to hit her. The bullet missed the manager but hit a customer, wounding him in the head.
In an action by the customer against the man for battery, the customer will be
(A) successful, because the man intended to shoot the manager.
(B) successful, because there was a “substantial certainty” that the customer would be hit by the bullet.
(C) unsuccessful, because the man could not foresee that the bullet would hit anyone other than the manager.
(D) unsuccessful, because the man did not intend to shoot the customer.
- (A) Under the doctrine of “transferred intent,” if the defendant shoots or strikes at A, intending to wound or kill him, but misses A and hits B instead, the defendant is held liable to B for battery. The intent to commit a battery to A is “transferred” or carried over to create liability to B. The man intended to shoot the manager, but he missed and, instead, hit a customer. The doctrine of transferred intent will enable the customer to hold the man liable for battery. Choice (B) is incorrect because it would be difficult in these circumstances to prove that the man knew the bullet was substantially certain to hit the customer. Given that the doctrine of transferred intent clearly applies, it is unnecessary for the customer to try to prove the man’s knowledge of a substantial certainty. Choice (C) is incorrect because it does not matter whether the man could foresee the bullet hitting someone other than the intended target. The doctrine of transferred intent applies, even if it was compLeteLy unforeseeable that the plaintiff would be hit. Choice (D) is incorrect because the doctrine of transferred intent supplies the intent required for the man’s customer’s battery claim.
- A taxicab driver stopped at a convenience store to buy a snack. While he was handing his money to the store clerk at the cash register, a police officer entered the store. The taxi driver had received several traffic tickets from the police officer, and he felt that the police officer had been very rude to him. Seeking to gain some revenge, the taxi driver immediately began to taunt and berate the police officer, calling him a variety of profane and insulting names. The police officer was going through a series of personal difficulties and was feeling very emotionally fragile. As the taxi driver’s insults rained down on him, the police officer suddenly snapped. He whipped out his gun and fired a shot at the taxi driver. The bullet missed the taxi driver but hit the store clerk, who was standing behind the taxi driver. The clerk survived the shooting but suffered a gunshot wound to his shoulder.
In an action by the injured clerk against the taxi driver to recover for his personal injuries, plaintiff will most likely
(A) recover, because the taxi driver’s conduct was the proximate cause of the clerk’s injuries.
(B) recover, because it was foreseeable that the taxi driver’s conduct would result in the clerk being shot.
(C) not recover, because the shooting was not a foreseeable consequence of the taxi driver’s conduct.
(D) not recover, because the police officer intended to shoot the taxi driver, not the store clerk.
- (C) The taxi driver did not intend for the shooting to occur, so an intentional tort claim would not be successful here. The plaintiff couLd claim that the taxi driver acted negligently, but will be unable to prove that the taxi driver’s conduct was a proximate cause of the shooting ard resulting injury. Proximate cause exists only if the plaintiff was within the general class of people that one could reasonably foresee being hurt, and the plaintiff’s injury was within the general type of harm that one could reasonably foresee occurring. Here, even if it was negligent for the taxi driver to insult and taunt the police officer, it was not foreseeable that the store clerk would be shot as a result of that negligence. In other words, no one could have foreseen that the police officer would snap and try to shoot someone who insulted him. Choice (A) is incorrect because, proximate cause is lacking here. Choice (B) is incorrect because the shooting was unforeseeable. Choice (D) is incorrect because it addresses the police officer’s intent, which would be very relevant for a cLaim against the police officer, but is not what will control whether the taxi driver can be held liable.
- An elderly woman was riding in an elevator from her apartment on the 12th floor to the lobby. When the elevator stopped on the fifth floor, a man entered the elevator smoking a cigar. The man was standing in front of the elderly woman on the elevator when the woman tapped him on the shoulder. When the man turned around, the woman pointed to the “No Smoking” sign and said, “Excuse me, sir, would you mind putting that cigar out?” The man indignantly responded by inhaling heavily on his cigar, and then he blew a big puff of smoke into the woman’s face. When the elevator stopped on the next floor, the man then departed.
If the woman brings a civil suit against the man for battery, who will prevail?
(A) The woman, because the smoke touched her face.
(B) The woman, because she had a reasonable basis for fearing that the man would attack her.
(C) The man, because he did not touch the woman.
(D) The man, because his conduct was annoying but did not inflict any bodily injury on the woman.
- (A) A person is subject to liability for battery if he acts intending to cause a harmful or offensive contact with the person of another. The essence of the plaintiff’s grievance is the offense to her dignity and invasion of the inviolability of her person, so a battery claim can arise even if the plaintiff suffers no physicaL harm to her body. In addition, the contact necessary for a battery claim can be accomplished directly or indirectly. For example, it can be a battery if someone hits you with his fist, but it also can be a battery if someone stabs you with a knife, throws a rock that strikes you, or fires a bullet that hits you. In addition, it can even be a battery if someone blows smoke in your face, if the contact between the smoke and your face is offensive. Choice (A), therefore, is correct, because the woman should be able to hold the man liable for battery. Choice (B) is incorrect because if the woman feared that the man might attack her, that would be a basis for an assault claim, not a battery claim. Choice (C) is incorrect because although the man did not directly touch the woman with his body, he caused the smoke to touch her, and that is sufficient to create a battery claim. Choice (D) is incorrect because battery can occur where a contact is offensive, not just where it is physically harmful.
- A woman lived in a town that has an ordinance that prohibits littering. While the woman was taking a stroll one morning, she passed a man who was standing on the sidewalk and eating a doughnut. As he finished the doughnut, the man wiped his mouth with a paper napkin. He then wadded up the napkin into a small ball and dropped it on the sidewalk. Seeing this, the woman stopped and admonished the man for throwing his trash on the ground. She told him about the town ordinance that prohibits littering and said that a police officer could give him a citation that would require him to pay a fine. The man was very insulted and annoyed to be lectured like this.
If the man institutes a civil action against the woman, the man will most likely
(A) recover for battery.
(B) recover for negligence.
(C) not recover, because the fact that an ordinance prohibited the man’s conduct would preclude recovery as a matter of law.
(D) not recover, because the woman’s conduct was customary and reasonable under the circumstances.
- (D) The man cannot satisfy the elements for any tort claim against the woman. Choice (A) is incorrect because a battery occurs when the defendant intentionally causes a harmful or offensive contact to another person. The man experienced no such harmful or offensive contact. Choice (B) is incorrect because negligence occurs when the defendant’s failure to exercise reasonable care causes harm to another person. The woman did not fail to act with reasonable care. Choice (C) is incorrect because the fact that the man violated the littering ordinance would not necessarily bar him from recovering if he otherwise could prove the elements of a tort claim. For example, if the woman had hit the man to punish him for littering, the man would be able to hold the woman liable for battery despite having violated the littering ordinance. By the process of elimination, that leaves only choice (0), which correctly states that the woman’s conduct was reasonable, and the man will not be able to hold her liable for any tort.
- A boat owner was sailing his boat at sea when a storm advisory was suddenly issued. The boat owner was navigating his craft toward shore when heavy rain and wind gusts began to rock the boat. Unable to reach his own slip, the boat owner docked his boat on a pier owned by a resident who lived in a nearby house.
The resident objected to the boat owner’s entry onto his private property. The boat owner explained that due to the storm it would be highly dangerous to be at sea. He requested permission to temporarily dock his boat until the storm subsided. The resident refused and cut the rope tying the boat to the dock. As a result, the boat drifted out to sea. The boat was battered by the rocky waves and high winds. Trying to prevent the boat from capsizing, the boat owner fell and broke his leg. The boat was extensively damaged in the storm, as well.
The boat owner has sued the resident, seeking to recover damages for his personal injury, as well as damage to the boat. The boat owner should
(A) recover for the damage to the boat, but not recover for his personal injury.
(B) recover for his personal injury, but not recover for the damage to the boat.
(C) recover both for damage to the boat and for his personal injury.
(D) not recover for either damage to the boat or his personal injury.
- (C) A person who trespasses on the property of another without a privilege can be held liable for doing so. On the other hand, a defender of property can be held liable for using force to prevent entry of someone who is actually privileged to intrude. In this question, the boat owner was privileged to dock his boat on the resident’s property because of the impending storm. Conversely, the resident was not entitled to prevent the boat owner from intruding upon his property. Therefore, the resident will be subject to liability for the boat owner’s personal injury, as well as for the damage to the boat. Choices (A), (B), and (D) are incorrect, as incomplete damages are given. Rememberthat when the defense of necessity is found to exist, damages are awarded for BOTH personal injuries and property loss.
- A man and a woman are adjoining homeowners in a residential neighborhood. The man begins to operate a slaughterhouse in his backyard. The putrid smell from the slaughterhouse travels over the woman’s property. The woman and her family members find the odor to be foul and nauseating. As a result of the man’s slaughterhouse, the value of the woman’s property was diminished by 20 percent, and the woman incurred expenses for prescription drugs to treat her family’s nausea. There is no applicable zoning ordinance preventing the man’s slaughterhouse operation.
The woman has asserted a nuisance action against the man. The complaint seeks recovery for damages and injunctive relief enjoining operation of the slaughterhouse. The court should
(A) award damages, but not grant injunctive relief.
(B) grant injunctive relief, but not award damages.
(C) grant injunctive relief and award damages.
(D) neither award damages nor grant injunctive relief, because there was no violation of any ordinance.
- (C) In an action for an injunction, the court may, in addition to or in lieu of granting an injunction, give one or more of the following remedies: (1) a judgment for damages; (2) a judgment for the recovery of property, or (3) a declaratory judgment. In order to avoid unnecessary litigation, courts having equity powers commonly award damages for torts committed prior to or pending suit, in addition to an injunction, in the same action. The plaintiff would not be required to make an election of remedies (i.e., to choose between seeking damages or injunctive relieO unless awarding both types of relief would be redundant and overcompensate the plaintiff. Choice (C) is thus the best answer because damages would be an appropriate remedy here to compensate the plaintiff for his financial loss. In addition, the court should also grant injunctive relief to prevent the nuisance from continuing in this residential neighborhood. Choices (A), (B), and (D) are incorrect because both injunctive relief and damages can be awarded.
- A rider entered a subway car at the 42nd Street station. Because all of the seats were occupied, the rider stood in the subway car and grabbed a pole to secure his balance. As the subway car was proceeding downtown, the rider glanced at a girl standing next to him. Suddenly, the subway car made an unexpected stop. The rider momentarily lost his balance and grabbed the girl around the waist (to avoid falling). Once the rider regained his balance, he removed his hands from the girl’s waist and grasped the pole again.
In a civil action instituted by the girl against the rider, he will most likely be found
(A) liable for battery.
(B) liable, because the rider believed that the girl consented to the contact under the
circumstances.
(C) not liable, because the rider’s conduct was socially acceptable under the circumstances.
(D) not liable, because the girl was not harmed by the contact.
- (C) Battery is an intentional tort in which the defendant causes harmful or offensive contact to another person. Whether contact is offensive is judged by an objective standard, meaning that it depends on what an average reasonable person would think. A contact is offensive if it would offend an ordinary person’s reasonable sense of personal dignity, and not if it would only be offensive to someone who is unduly sensitive. Thus, the contact must be unwarranted by the social usages prevalent at the time and place at which it is inflicted. Under the circumstances presented here, the rider’s conduct would be socially acceptable. He intended to touch the girl, but only to prevent himself from falling. He, therefore, would not be liable for battery. Choice (A) is incorrect because the offensive or harmful contact required for a battery is not present. Choice (B) is incorrect because the rider does not even need to raise the issue of whether the girL consented or if he thought she consented. Consent is an affirmative defense to an intentional tort claim, but the case will not get that far because the eLements of a battery claim cannot be established by the plaintiff. Choice (D) is incorrect because a battery requires contact that is either harmful or offensive. The fact that the girl was not harmed here, therefore, does not, by itself, prevent Liability from being imposed. It is the fact that the contact was neither harmful nor offensive that is crucial.
A fortune teller told fortunes by means of Tarot cards. An elderly woman, who was worried about her failing health, had heard that the fortuneteller was clairvoyant and could see into the future. Consequently, the woman decided to see the fortuneteller in order to have her Tarot cards read. As the fortuneteller was telling the woman her fortune, she suddenly said, “I have a vision. If you give me $25,000 tomorrow, you will live to be 100 years old.” The woman, who was 72 years of age, believed the fortuneteller and gave her the money the next day. The following week, the woman’s physician informed her that she had a serious heart disease and he didn’t expect her to live for more than a year or two.
If the woman asserts a claim against the fortuneteller based on deceit, the plaintiff should
(A) prevail, because she relied to her detriment on the fortune teller’s foretelling.
(B) prevail, if the fortuneteller did not honestly believe that the woman would live to be 100 years of age.
(C) not prevail, unless there was a fiduciary relationship between the parties.
(D) not prevail, unless the fortuneteller warranted the truth as believed.
- (B) The tort of deceit (also known as intentional misrepresentation or fraudulent misrepresentation) requires proof that a faLse representation has been made (1) knowingly, or (2) without belief in its truth. The intent that becomes important is the intent to deceive, to mislead, or to convey a false impression. This intent, which has been given the name “scienter” by the courts, must be a matter of belief, or an absence of belief, that the representation is true. Certainly, there is no difficulty in finding the required intentto mislead where it appears that the speaker believes his statement is false. Choice (A) is incorrect because reliance would not be sufficient to create liability if the scienter requirement is not satisfied. Choice (C) is incorrect because a deceit action requires that a fiduciary relationship exist between the plaintiff and defendant. Choice (D) is incorrect because if the fortuneteller warranted the truth, then no action for deceit would be present.
While relaxing at poolside one Sunday afternoon, a homeowner was struck by a golf ball driven by a 14-year-old boy who was playing the ninth hole at the local golf course. The fairway for the ninth hole was 65 feet wide and 437 yards long, with a dogleg in an easterly direction. Between the fairway and the homeowner’s property was a “rough,” containing brush and low-lying trees. The boy had hit a towering shot down the middle of the fairway that deflected off a tree and struck the homeowner in the head.
The homeowner brought suit against the boy for his injuries suffered when he was hit by the golf ball. At trial, the boy offered uncontested evidence that golf balls from the golf course regularly traversed onto the homeowner’s property two to three times a day.
Which of the following statements is most accurate regarding the boy’s liability for trespass?
(A) The boy is not liable, because he did not inten
tionally cause the golf ball to traverse onto the
plaintiff’s property.
(B) The boy would be liable for the unpermitted
intrusion of the golf ball onto the plaintiff’s
property.
(C) Because the plaintiff should have reasonably
anticipated that living next to a golf course
would result in stray golf balls landing on his
property, the boy would not be held liable.
(D) Because the golf ball did not substantially
interfere with the plaintiff’s use and enjoyment
of his land, the boy would not be held liable.
- (A) The intentional tort of trespass to land occurs where the defendant intentionally causes someone or something to enter or to remain on land of another person. A defendant can also be held liable for negligently causing a trespass to land that injures someone. Here, the boy caused the golf ball to enter the homeowner’s property, but the facts do not indicate that this was intentional or negligent. Choice (B) is incorrect because liability will not exist without proof of intent or negligence. Choice (C) is incorrect because liability could exist, even if the entry of the golf ball was foreseeable, if it was done intentionally or negligently. Choice (D) is incorrect because a negligence or intentional trespass to land claim would not require proof of substantial interference with the plaintiff’s use and enjoyment of his land. That language is associated with the tort of private nuisance, not negligence or trespass.
- After watching a television program about archery, a 15-year-old boy became very interested in that sport. He saved up some money to buy a bow and other archery equipment. He set up a target in his backyard in order to practice. He surrounded the target with stacks of hay bales to stop any arrows that missed the target. After practicing for a few weeks, the boy made great improvements in his technique and accuracy. While practicing one afternoon, however, the boy lost his balance just as he released an arrow. As a result, the arrow flew way off course, going over the target and all the hay bales and flying into a wooded area behind the boy’s house. The boy assumed no one was in the woods, so he was relieved that the errant arrow would not hurt anyone. However, a hiker happened to be in the woods near the boy’s house. As he paused for a moment to take a drink of water, the hiker suddenly saw something out of the corner of his eye. It was the stray arrow shot by the boy. Without even thinking about it, the hiker reflexively ducked out of the way. The arrow narrowly missed the hiker’s head; but as the hiker ducked out of the way, his head struck the limb of a tree, causing a stick to stab painfully into his eye.
Which of the following causes of action could the hiker successfully assert against the boy?
(A) Assault, but not battery.
(B) Battery, but not assault.
(C) Assault and battery.
(D) Neither assault nor battery.
- (D) No intentional tort occurred here. Battery requires proof that the defendant did an act with intent to cause a harmful or offensive contact to another person. Assault requires proof that the defendant intended to put another person in apprehension of imminent harmful or offensive contact. For purposes of intentional torts, “intent” means that the defendant had a desire or purpose for a certain result to occur or that the defendant knew with substantial certainty that the result would occur. Here, the boy did not desire to hit anyone with an arrow or to put anyone in apprehension of being hit. He also did not have knowledge of a substantial certainty that someone would be hit. Choices (A), (B), and (C), therefore, are incorrect because no battery or assault occurred.
A 12-year-old girl and her friends were playing catch with a baseball in the middle of a field of grass in the park. Near the edge of the field, a woman was sitting in a beach chair and reading a book. The girl threw the ball too far, and it went over her friends’ heads and flew toward the woman. Although the woman did not see the ball coming, it hit the straw hat that the woman was wearing, knocking it from her head. Although the woman was not touched by the ball, she was startled by the ball hitting her hat, and she fell from her chair and broke her arm.
If the woman initiates a suit against the girl to recover damages for her broken arm, the woman will
(A) recover for assault only.
(B) recover for battery only.
(C) recover for assault and battery.
(D) not recover.
- (D) Intent is a crucial element of intentional torts, including battery and assault. Battery requires proof that the defendant did an act with intent to cause a harmful or offensive contact to another person. Assault requires proof that the defendant intended to put another person in apprehension of imminent harmful or offensive contact. For purposes of intentional torts, “intent” means that the defendant had a desire or purpose for a certain result to occur or that the defendant knew with substantial certainty that the result would occur. Here, the girl did not have the required intent. Choices (A), (B), and (C), therefore, are incorrect because nO battery or assault occurred.
A sportsman was the owner of an old dilapidated stadium, which was located on the outskirts of the city. The stadium, which was built in 1932, had been the home stadium for the local professional baseball team for 30 years. However, in 1962, the baseball team franchise moved to another city. Since 1962, the stadium was left unattended and had deteriorated to such an extent that the walls were in danger of collapsing.
Last month, an earthquake struck the city. The earthquake, which registered 6.9 on the Richter scale, caused considerable damage in the city and caused the stadium to collapse. As the stadium crumbled to the ground, a large section of the press box fell on top of a car that was parked nearby. The auto was crushed, causing its gas tank to rupture. As a result, a large quantity of gasoline spilled along the street and flowed downhill. The gasoline collected in front of a homeowner’s home, which was located about a mile from the stadium.
Two hours after the earthquake struck, a pedestrian was walking in front of the homeowner’s home, smoking a cigarette. When he discarded his lighted cigarette butt in the street, the gasoline exploded. The explosion blew the windows out of the homeowner’s home. The homeowner, who was sitting in the living room watching television, was struck by the flying glass and injured.
If the homeowner asserts a claim for his injuries against the sportsman, which of the following is the sportsman’s best defense?
(A) The earthquake was an act of God.
(B) The sportsman’s negligence, if any, merely created a passive condition and was not the active
cause of the homeowner’s injury.
(C) The sportsman could not reasonably have been
expected to foresee injury to a person in the
homeowner’s position.
(D) The pedestrian’s act of discarding the lighted
cigarette in the street, which sparked the explosion, was the proximate cause of the homeowner’s injury.
- (C) Foreseeability and proximate causation are specific areas that are highly tested. Students must be able to analyze multiple negligent acts that follow directly (i.e., an uninterrupted chain of events) orindirectly (i.e., where a period of time passes between intervening events) from the initial negligent conduct of the defendant, and then determine, by applying a test of foreseeability, whether or not the defendant’s initial conduct remains as the proximate, or legal, cause of the plaintiff’s harm. In this question, it must be determined whether the homeowner, the plaintiff, was a foreseeable plaintiff to whom the sportsman owed a duty. According to the majority view, a defendant owes a duty of care only to those plaintiffs who are foreseeably within the risk of harm created by the defendant’s conduct (i.e., within the “zone of danger”). If the sportsman could not reasonably have been expected to foresee injury to a person in the homeowner’s position, then no duty would be owed and, therefore, the sportsman’s negligence could not be the proximate cause of the homeowner’s harm. Here, when the sportsman negligently failed to keep the stadium in safe condition, he could have reasonably foreseen that an earthquake might cause it to collapse and injure someone who was in the stadium or next to it. He could not have reasonably foreseen that the condition of the stadium would cause someone to be injured, like the homeowner, who was watching television in his house a mile away. Choice (A) is incorrect because a defendant can be held liable for foreseeable consequences of negligence even if the particular manner in which the harm occurred involved an unexpected natural force, like an earthquake or weather. Choice (B) is incorrect because a defendant can be Liable even if its negligence is a “passive” rather than an “active” cause of the plaintiff’s harm. Choice (D) is incorrect because an injury may have more than one proximate cause. The question is whether the sportsman’s negligence was “a” proximate cause of the plaintiff’s injury, not whether it was “the” proximate cause. The fact that the pedestrian was the most proximate or direct cause of the injury, therefore, would not necessarily preclude the sportsman from being held Liable.
A tall building was under construction in the downtown business district of a city. A lawyer drove her car to the downtown area to go to a meeting with a client. She was running late for the meeting and could not find a legal parking spot, so she decided to park illegally in front of a fire hydrant. This was in violation of a local ordinance that prohibited parking within 50 feet of a fire hydrant. The lawyer figured that it was better to get a ticket than to miss her meeting with an important client. The spot where she parked was next to the site of the construction of the new building.
While the lawyer was at her meeting, an accident occurred at the construction site. A large crane was being used to lift a load of bricks. A cable on the crane broke, and the bricks fell. Most of them landed on top of the lawyer’s car. The load of bricks that fell on the lawyer’s car caused the gas tank of the car to rupture and explode. Shrapnel from the explosion flew in all directions and injured a pedestrian who was talking on the sidewalk near the lawyer’s car.
If the pedestrian sues the lawyer and relies on the doctrine of negligence per Se, which of the following, if true, is the lawyer’s best defense?
(A) Payment of a small fine is the only penalty provided in the ordinance for those who park too close to fire hydrants.
(B) The police never issued a ticket to the lawyer for parking in front of the fire hydrant.
(C) The purpose. of the parking ordinance was to
facilitate access to the hydrant by fire trucks,
not to protect against accidents like the one
that occurred when bricks fell on the lawyer’s
car.
(D) The pedestrian would not have been injured if
the construction company had properly maintained the crane.
- (C) For a negligence claim, the jury usually must assess all the facts and circumstances to decide whether the defendant acted with as much care as a reasonable person. In most states, however, the doctrine of “negligence per Se” provides a shortcut to proving that the defendant failed to exercise reasonable care. A defendant who violated a statutory standard of care will be treated as negligent per Se, meaning that the defendant’s conduct will automatically be deemed to be negligent. Breach of a statutory duty will be negligence per se only if two requirements are met: (1) the plaintiff must be a member of the class of persons meant to be protected by the statute; and (2) the plaintiff’s injury must be of the type of harm the statute was designed to prevent. Here, if the defendant lawyer can show that the plaintiff pedestrian’s injuries were not of the type intended to be prevented by enactment of an ordinance prohibiting parking within 50 feet of a fire hydrant, she will not be negligent perse. Therefore, choice (C) is correct. Exam Tip: A small minority of jurisdictions follow the rule that violation of a criminal statute is merely evidence of negligence, but not negligence per se. Students should follow the majority rule on the MBE, unless the facts say otherwise. Choice (A) is incorrect because negligence per se can apply even if the criminal penalty for violating a statute is only a minor fine. Choice (B) is incorrect because negligence per se would not require proof that the lawyer received a ticket for violating the ordinance. Choice (D) is incorrect because the lawyer’s liability would not depend on whether the construction company was negligent. In other words, even if the construction company was negligent and that was a cause of the accident, that would not necessarily preclude the lawyer from being heLd liable, as well.
A wife and her husband were having dinner at a restaurant when the wife excused herself to go to the bathroom. The restaurant was owned and operated by a chef. As the wife was walking past a table where another customer was seated, she slipped and fell on an egg roll that had been lying on the floor for quite some time, although the chef was unaware that it had fallen onto the floor. When she fell, her head struck a serving tray that was located in the aisle. The fall caused the wife to suffer a severe concussion. The customer knew that the egg roll was on the floor, and although he could have done so, he did not warn the wife.
If the wife asserts a claim against the chef for the injuries she suffered from the fall, she will most likely
(A) recover, because the egg roll on the floor constituted an unsafe condition of the premises.
(B) recover, because the egg roll was on the floor for a substantial period of time before the accident.
(C) not recover, because the chef did not know that the egg roll was on the floor.
(D) not recover, because the customer could have prevented the injury by warning the wife of the presences of the egg roll.
- (B) A person who possesses Land generally owes a duty to exercise reasonable care for the safety of invitees. An invitee is someone who enters land that is held open to the general public or who enters the land in connection with some potential business dealing that would benefit the person who possesses the land. The obligation to exercise reasonable care extends to everything that threatens the invitee with an unreasonable risk of harm—care against negligent activities, warning of known latent dangers, precautions against foreseeable dangers, and inspection of the premises to discover possible dangerous conditions that are not known. As a patron in a restaurant, the wife should be classified as an invitee. The chef thus owes a duty to exercise reasonable care for her safety. In this very tricky Torts question, students must choose the answer choice that is the most precisely correct and is neither over-inclusive nor under-inclusive. Choice (B) is correct because the chef has a duty to act reasonably in inspecting the premises, and that includes inspecting to discover an unsafe condition within a reasonable period of time. Therefore, because the egg roll had been on the floor for a substantial period of time, the chef would be liable. Choice (A) is incorrect because a business like a restaurant cannot be expected to know immediately about every unsafe condition, such as a piece of food on the floor. Choice (C) would be correct if the wife were merely a licensee, because then the property owner would merely have a duty to warn her about known dangers. She is an invitee, however, so the duty owed to her is broader and includes warning her about dangerous conditions the owner either knew or should have known about through reasonable inspection. Choice (D) is incorrect because an omission to act by a third person does not relieve a defendant of liability.
The head coach of the local high school football team was also employed as a physical education teacher at the high school. It was a late autunm afternoon, and the football team was practicing for its upcoming game against its archrival. While the team was practicing, the skies darkened as a storm approached. Minutes later, it began to rain heavily.
Even though the storm intensified, the coach did not stop the practice because he had no reason to believe the storm presented a danger to his players. Suddenly, a bolt of lightning struck and killed the team’s star player. This jurisdiction has abolished governmental immunity, and a school board may be sued as a private entity.
If the player’s parents bring a wrongful death action against the coach and the high school board, will they prevail?
(A) Yes, because the player was injured while engaged in a school activity.
(B) Yes, because under the circumstances the coach would be strictly liable.
(C) No, because the coach was not negligent by continuing the practice.
(D) No, because lightning is an act of God.
- (C) Although the player’s death was an accident, the coach (and the school board) may be held liable for the wrongful death if it were negligent for the coach to continue practice in spite of the storm. However, the facts state that the coach had “no reason to believe the storm presented a danger to his players,” and that the lightning came “suddenLy” out of what was otherwise a storm involving only rain. If he had no reason to know of the danger, he did not act unreasonably and, therefore, was not negligent. Choice (A) is incorrect; even though the player was, in fact, injured during a school activity, since the coach was not negligent, there would be no liability. Choice (B) is incorrect because the coach was not engaged in a type of abnormally dangerous activity that would give rise to strict liability. Those types of behavior would include such activities as blasting, storage/transportation of dangerous chemicals or explosives, or selling a defective product. Because the coach was simply coaching his team, strict liability will not attach. Choice (D) is incorrect for two reasons. First, an intervening act of God does not cut off liability unless it is unforeseeable, and there is no showing that lightning was unforeseeable in the area. Second, since the coach was not negligent, causation does not matter.
- A husband and wife were shopping at a local department store when the wife saw a dress that she especially liked. The wife, who weighed more than 300 pounds, was unable to find a size large enough to fit her. She then saw a store clerk and asked if the store carried the dress in her size. The clerk looked at the wife and said, “You look like a hippopotamus, and I’m sorry, but we don’t carry this dress in the hippo size.” Another customer in the store overheard the comment and began to laugh. The wife became very upset and hurried out of the store.
If the wife asserts a tort action against the department store based upon the clerk’s actions, the wife will
(A) win, because the statement was overheard by
another customer.
(B) win, because the clerk’s conduct was extreme and outrageous.
(C) lose, because the clerk was merely statingan opinion.
(D) lose, because the wife only suffered hurt feelings.
- (D) In order to recover for slander, the plaintiff must plead and prove special damages (i.e., pecuniary loss). To recover for infliction of emotional distress, the plaintiff must suffer severe emotional distress. Choice (D) is, therefore, correct because the wife would not have a claim if she suffered only hurt feelings. Choices (A) and (C) are incorrect because they relate to the possibility of the defendant being held liable for defamation, but the wife cannot recover on such a claim without sufficient damages. Likewise, choice (B) is incorrect because it suggests that the wife could recover for infLiction of emotional distress, but that would not be possible unless the wife suffered severe emotional distress.
- On Thanksgiving Day, a father was carving a turkey for his family when he seriously cut his hand. The knife severed an artery, causing the father to lose a lot of blood. The mother telephoned their family doctor, who instructed her to drive the father to the local hospital. The doctor indicated that he would meet them there.
A short time later at the hospital, the doctor treated the father and bandaged his hand. However, due to the fact that the father had lost a considerable amount of blood, the doctor ordered an immediate blood transfusion. After determining that the father’s blood type was A-positive, the doctor orally instructed a nurse, who was employed by the hospital, to obtain the necessary blood for the transfusion. The nurse followed the doctor’s instructions and telephoned the hospital’s blood bank department and ordered a unit of A-positive blood.
The person in the blood bank who received the nurse’s call correctly wrote down the order for A-positive blood. However, someone working at the blood bank, whose identity cannot be determined, made an error in filling the order and labeling the unit of blood for the father. As a result, the blood bank sent a unit of B-positive blood mislabeled as A-positive blood. After being administered the wrong blood, the father died.
The hospital has a written rule in effect whereby all orders for blood transfusions must be made in writing by the physician in charge of the patient’s treatment. Although the doctor was aware of the hospital rule, he, like many other doctors at the hospital, frequently ordered blood transfusions by oral instructions.
If the mother asserts a wrongful death action against the doctor, she will most likely
(A) prevail, because the doctor would be responsible for the blood bank’s error in administering the wrong blood.
(B) prevail, because the doctor did not follow the hospital’s rule regarding blood transfusion orders.
(C) not prevail, because the doctor acted in the same fashion as many other doctors at the hospital.
(D) not prevail, because the unidentified person in the blood bank who was responsible for sending the wrong blood type was not an employee of the doctor.
- (D) Under the doctrine of respondeat superior, a master is generaLLy hetd vicariously liable for aLl tortious conduct of his servant that is within the “scope of employment.” The blood bank employee (who mislabeled the blood) was not an employee or servant of the doctor. Therefore, the doctor would not be responsible for that negligence. Choice (A) is incorrect because the doctor will not be held liable because the doctrine of respondeat superior does not apply. Choice (B) is incorrect because the doctor’s failure to submit the blood transfusion order was not an actual cause or a “but for” cause of the patient’s death. The blood bank received the correct order (for A-positive blood) from the nurse. Despite having received the order correctLy, the blood bank then sent the wrong blood, which is an error that would have occurred regardless of whether the doctor submitted the order in writing or orally. The patient thus would have died because of the blood bank’s error, even if the doctor had submitted the order in writing and, therefore, his violation of the hospital rules was not a cause of the patient’s death. Choice (C) is incorrect because the fact that the doctor did what many other doctors had done would not necessarily shield him from liability. However, as stated above, the doctor was not the actual cause of the father’s injury, so it does not matter whether the doctor acted negligently.
- A patient was in the hospital to be treated for an illness. The patient’s doctor prescribed a mild anesthetic drug, to be given to the patient intravenously, to reduce the pain being experienced by the patient. The patient was supposed to receive a saline solution containing 18.5 milligrams of the drug. A nurse arranged to have a pharmacy technician prepare the solution and drug, and then the nurse administered the solution and drug to the patient. The solution that was administered to the patient contained 185 milligrams of the drug, rather than just 18.5. Shortly after receiving the drug, the patient had a heart attack and died. The doctor, nurse, and pharmacy technician were all employed by the hospital.
If the patient’s family brings a wrongful death claim against the hospital, which of the following must the family prove in order to recover damages?
(A) The family must identif’ which of the three hospital employees involved—the doctor, nurse, and pharmacy technician—was actually responsible for the mistake in the dosage of the drug.
(B) The family must prove the specific negligent act that resulted in the wrong dose of the drug being administered.
(C) The family must show that hospital administrators were negligent in hiring or supervising one or more of the employees involved.
(D) The family must prove that the patient would not have died if given 18.5 milligrams of the drug.
- (D) The doctrines of respondeat superior and res ipso loquitur can be used together in this situation. Respondeat superior means that an employer will be vicariously Liable for torts committed by its employees within the scope of their employment. Res ipso Ioquitur means thata plaintiff can recover on a negligence claim, even if he or she is unable to prove exactly what was done negligently, if the plaintiff can show that the accident in which the plaintiff was injured is the sort of thing that ordinariLy does not occur unless there was negligence, and the agency or instrumentality that caused the injury was within the exclusive control of the defendant. In such a situation, res ipsa loquitur permits the jury to infer, based on the circumstantial evidence, that the defendant was negligent, even if the plaintiff cannot prove exactly what negligent act occurred. In this question, the hospital is responsible under respondeatsuperior for any torts of the doctor, nurse, or physician-technician. Even if the plainfiffs cannot prove exactly what happened that was negligent, the plaintiffs can rely on res ipsa loquitur because a patient ordinarily does not receive the wrong dose of medication unless someone was negligent. Choice (A) is incorrect because it is not necessary to show the identity of a particular employee of the hospital. There is an inference that it was an employee, and that is enough. Choice (B) is incorrect because res ipsa loquitur will allow the plaintiffs to recover through an inference of negligence even if they cannot determine exactly what negligent act occurred. Choice (C) is incorrect because the family can hold the hospital liable for the tortious conduct of one or more of the employees, even without proof that the hospital was negligent in hiring or supervising any of the employees. Choice (D) is correct because the plaintiffs do need to prove that the dosage error was an actual cause or a “but for.” cause of the patient’s death. In other words, they must prove that the patient would have lived if given the correct dose.
One fall weekend, an outdoorsman went on a camping trip to a remote wilderness area deep in the northern part of the state in which he lived. While on the trip, the outdoorsman went deer hunting. He was in a very desolate area, surrounded by a densely wooded terrain, that was located at least 100 miles from any habitation. After a rather uneventful morning with nothing at which to shoot, the outdoorsman suddenly spotted a bald eagle; a nearly extinct bird. The bald eagle was listed as an endangered species, and to shoot one in this state was a criminal offense.
Unable to resist the temptation, the outdoorsman took a shot at the bald eagle. The bullet missed the bald eagle but struck a hermit, who had moved to the woods a few months earlier to escape from the stresses of society. The hermit had been napping in a secluded area. The bullet hit the hermit in the eye and permanently blinded him. The outdoorsman was unaware of the hermit’s presence.
If the hermit asserts a claim against the outdoorsman to recover damages for his injury, the hermit will
(A) prevail, because his injury was caused by the
outdoorsman’s unlawful act.
(B) prevail, because firearms are dangerous instru
mentalities imposing strict liability on the user.
(C) not prevail, because the outdoorsman had no
reason to anticipate the presence of another
person in such a remote area.
(D) not prevail, because the outdoorsman did not
intend to shoot the hermit.
- (C) A popular Multistate testing area deals with proximate, or legal, cause. In order to be liable for negligence, the defendant’s conduct must constitute the legal or proximate cause of the plaintiffs harm or injury. The majority view is that a defendant’s duty of care is owed only to foreseeable plaintiffs (i.e., those individuals who are within the risk of harm created by the defendant’s unreasonable conduct). Choice (C) is the best answer because it addresses the fact that the hermit was an unforeseeable plaintiff to whom no duty of care was owed. Choice (A) is incorrect because the statute prohibiting people from shooting bald eagles was meant to protect the bald eagles, not to protect a person like the hermit, who was accidentally hit. Choice (B) is incorrect because there is no basis for imposing strict liability here. Moreover, even a strict liability claim would require that the plaintiff was a foreseeable person, and the hermit was not a foreseeable victim of the defendant’s conduct here. Choice (D) is incorrect because the outdoorsman’s lack of intent would prevent him from being held liable for intentional torts, but would not help him to avoid liability for negligence.
- A hunter sat quietly in a forest, hiding behind some brush and waiting for deer to pass by him. After hours of waiting and not spotting any deer, the hunter decided to give up and go home. Disappointed that he never even got a chance to take a shot at a deer, the hunter decided to fire a few practice shots before walking back to his car. He aimed his rifle at a small tree at the top of a nearby hill and fired three shots.
At that moment, a state police officer happened to be passing nearby. The officer was part of a police team searching the forest for a fugitive who was believed to be armed and dangerous. When the police officer heard the gunshots, he thought he was being attacked. He quickly pulled his pistol out of his holster and fired it toward the brush from which he had seen the muzzle flash from the hunter’s rifle. One of the bullets fired by the police officer struck the hunter and seriously wounded him.
If the hunter asserts a claim against the police officer to recover damages for his injury, the hunter will
(A) prevail, because the hunter had no intent to harm the police officer and, therefore, the police officer was not entitled to fire back.
(B) prevail, because the hunter had not committed any crime entitling the police officer to use force likely to cause death or serious bodily injury.
(C) not prevail, if the police officer reasonably believed that he was under attack.
(D) not prevail, because the police officer was not the original aggressor.
- (C) Self-defense is one of the defenses that can be used to avoid liability for battery or other intentional torts. The key requirement is reasonableness. The person who acts in self-defense must reasonably believe that he is in danger and must use no more than a reasonable amount or degree of force under the circumstances. Furthermore, a reasonable mistake as to the existence of the danger does not vitiate the defense. In other words, what matters is whether it reasonably appeared that self-defense was necessary, not whether it was actually necessary. Even though the police officer was mistaken when he thought he was under attack, he would have a valid defense if it reasonably appeared to him that he was in danger and needed to defend himself by shooting back at the attacker. Choice (A) is incorrect because even though the hunter did not intend to shoot anyone, the police officer still was entitled to act in self-defense if he reasonably believed he was under attack. Choice (B) is incorrect because if the police officer reasonably believed that he was in danger and needed to protect himself, he would be entitled to shoot, regardless of what crimes, if any, the attacker may have previously committed. Choice (D) is incorrect because even if the police officer was not the original aggressor, he would be entitled to use self-defense if he reasonably believed it was necessary to protect him from danger.
- A pregnant woman was sitting on her front porch watching her husband mow the lawn. A friend of the woman had known the woman for years and knew that she was an easily excitable person. The friend also hated the woman’s husband. Knowing that the woman was present, the friend drew a pistol and threatened to kill the husband. The woman suffered severe emotional distress as a result of witnessing this incident and soon afterward had a miscarriage.
In an action by the woman against the friend for mental anguish resulting in her miscarriage, the woman will
(A) lose, because the friend did not intend for the woman to suffer a miscarriage.
(B) lose, because the friend’s actions were directed against the husband, not the woman.
(C) win, because it was highly probable that the friend’s extreme and outrageous conduct would cause emotional distress to the woman.
(D) win, because she is the husband’s wife.
- (C) A person is liable for intentional infliction of emotional distress if he does something that is extreme and outrageous and that intentionally or recklessly causes severe emotional distress to someone. In general, only the person who is the target of the extreme and outrageous conduct can recover. However, a plaintiff can recover if she was present when the defendant did something extreme and outrageous to the plaintiff’s immediate family member, and if the defendant knew or should have known that defendant was present and likely to suffer severe emotional distress from witnessing the event. Here, the woman suffered severe emotionaL distress because she was present when the friend did something extreme and outrageous to her husband. She, therefore, can hold the friend liable because the friend knew or should have known she would suffer such emotional distress, since he knew she was an easily excitable person. Choice (A) is incorrect because it is not essential that the defendant intended that the woman suffer a miscarriage, provided that he did know or have reason to know she would suffer severe emotional distress from witnessing the threat to her husband’s life. Choice (B) is incorrect because the woman could recover, even though she was not the target of the friend’s threat, through the exception for family members described above. Choice (D) is incorrect because it is not enough that the woman is the husband’s wife. Alt of the other requirements described above must be satisfied, including that the friend knew or should have known that the woman would suffer severe emotional distress. Choice (C) is a better answer than (D) because choice (C) provides the additional fact that the friend should have realized his extreme and outrageous conduct would cause emotional distress to the woman.
- A child was playing with a ball in the front yard of his family’s home. He accidentally threw the ball too far, and it hit a neighbor’s new car. The neighbor was enraged and threatened to kill the child. The child ran inside his house and told his mother what had occurred. Although the child was only mildly upset by the incident, the child’s mother was extremely distressed. She suffered an emotional breakdown and needed months of therapy to recover.
If the mother sues the neighbor for the mental anguish suffered, she will
(A) recover, even though she was not present when the threat was made.
(B) recover, because she would not have suffered the emotional trauma had it not been for the neighbor’s threat to her child’s life.
(C) not recover, because she was not present when the threat was made.
(D) not recover, because the neighbor did not touch her child.
- (C) Where a defendant’s extreme and outrageous conduct is directed at a third person, rather than at the plaintiff herself, the plaintiff generally cannot recover for intentional infliction of emotional distress. An exception to that general rule applies where the plaintiff is present when the extreme and outrageous conduct occurs, the target of the extreme and outrageous act is an immediate family member of the plaintiff, and the defendant knows or should know the plaintiff is present and will suffer severe emotional distress from witnessing what happens to her family member. That exception does not apply here because the mother was not present when the threat was made to her child. Choice (A) is incorrect because in order for the mother to be successful, she will have to show that she was present at the time of the confrontation between the neighbor and her child. Choice (B) is incorrect because she cannot recover for her emotional distress, even if hearing about the neighbor’s conduct caused that distress, because she was not present when the threat to her child was made. Choice (D) is incorrect because if she had been present when the threat was made, she could be able to recover for intentional infliction of emotional distress, even though the neighbor did not touch her child.
- A prominent judge lived next door to a father. Recently, the judge had sentenced the father’s son to six months in prison on a narcotics charge. One afternoon while the judge was mowing his lawn, the father decided to avenge his son’s conviction. The father set up his water sprinkler behind some shrubbery separating their adjoining properties. As the judge was mowing his lawn and came within reach of the water sprinkler, the father turned on the sprinkling device. The judge did not see the water coming toward him. The water hit the judge in the back and drenched him.
The judge would be able to recover against the father for which of the following tort or torts?
(A) Negligence and battery.
(B) Battery and assault.
(C) Negligence and assault.
(D) Battery and trespass to land.
- (D) A person is liable for battery if he does an act with the intent and result of causing harmful or offensive contact to the plaintiff. A person is liable for trespass to land if he does an act with the intent and result of causing someone or something to enter or remain on someone’s land. Because the father intended to douse the judge with water (from the sprinkling device), he would be liable for a battery. In addition, by directing water over the judge’s property, the father would be liable for trespass. Choice (A) is incorrect because negLigence is the wrong cause of action when the defendant acts intentionally. Choice (B) is incorrect because assault requires that someone was intentionally put in apprehension of an imminent harmful or offensive contact. Here, the judge did not see the water coming, so he was not put in apprehension of being hit by it. Choice (C) is incorrect because neither negligence nor assault is an appropriate claim here, for the reasons just given.
- Each year, the local boys’ and girls’ club sponsors a “Baseball Card Bonanza.” During the three- day event, collectors of baseball cards gather, exchanging and swapping old cards. A boy and a girl, both 12-year-old youngsters at the bonanza, were looking at each other’s cards when they were approached by a 32-year-old collector. The collector asked the boy and girl if he could look through their baseball cards. The boy and girl nodded affirmatively.
As the collector was flipping through the boy’s cards, he found a 1948 card featuring Branch Rickey, the former owner of the Brooklyn Dodgers. The collector, who knew the card was worth more than $500, offered the boy 50 cents for the card. The boy thought the card had very little value because he knew that Rickey was not a major league baseball player. The boy accepted the 50 cents and gave the Rickey baseball card to the collector. The next day the boy found out that the baseball card was worth
$600.
If the boy asserts a claim against the collector for deceit, will the boy prevail?
(A) Yes, because the collector did not disclose the true value of the card.
(B) Yes, because the boy was the true owner of the card and was entitled to the benefit of the bargain.
(C) No, because the collector made no false representations of fact.
(D) No, because the boy was not justified in relying on the collector’s offer.
- (C) The elements of the tort action for deceit may be summarized as follows: (1) a false representation made by the defendant (usually this representation must be one of fact); (2) knowledge or belief on the part of the defendant that the representation is false; (3) an intention to induce the plaintiff to act or to refrain from action in reliance upon the misrepresentation; (4) justifiable reliance upon the representation on the part of the plaintiff; and (5) damage to the plaintiff, resulting from such reliance. In the present hypothetical, because the defendant did not make any false representation of fact, the correct answer is choice (C). Choice (A) is incorrect because non-disclosure of value does not create liability for deceit. Choice (B) is incorrect because “benefit of the bargain” speaks to the potential recovery of the plaintiff should the tort of deceit be established. Since the boy cannot establish that a deceit took place, he is not entitled to this remedy. Choice CD) is incorrect because justifiable reliance alone is not enough to show deceit. The plaintiff must prove that the defendant made a false representation of fact.
On Tuesday, a homeowner purchased a new riding lawnmower from a department store. The next day, he was cutting the grass in his back yard with the mower when it started to rain. He stopped the mower and turned off the motor switch. He went inside and decided to wait until the rain stopped before mowing the rest of the lawn. A few minutes later while the homeowner was inside his house, the lawnmower suddenly lurched forward, rolled down a hill, and entered the property of his neighbor. The mower cut the neighbor’s prize rosebushes, destroying them.
It was later determined that the lawnmower’s motor switch was defective. Even though the homeowner had turned the starter switch to the off position, the engine did not shut off. Because the mower was built with a new silent rotary engine, the homeowner had no reason to know that it was still running when he went into the house. The homeowner would not have been able to discover the defect by any feasible means of inspection.
If the neighbor asserts a claim against the homeowner for trespass, will the neighbor prevail?
(A) Yes, because the lawnmower entered onto the neighbor’s property.
(B) Yes, because the homeowner is strictly liable for the damage caused by the defective mower.
(C) No, because the homeowner was using the mower for its intended purpose.
(D) No, because the defective motor switch was not discoverable by reasonable inspection.
- (D) There are several theories through which a plaintiff could hold a defendant liable for something (like the defendant’s lawnmower) entering the plaintiff’s land. One is the intentional tort of trespass to land, which requires proof that the defendant intended for the thing to enter the plaintiffs land. Another would be a negligence claim if the defendant’s failure to exercise reasonable care caused the thingto enter the plaintiff’s land. The defendant also could be strictly liable if his abnormally dangerous or ultrahazardous activities caused the thing to enter the plaintiff’s land. However, if none of those theories applies, the defendant will not be liable. Here, the lawnmower’s entry onto the neighbor’s land was not the result of any intentional act, negligent conduct, or abnormally dangerous activity by the homeowner. Choice (A) is incorrect because the entry here was accidental and not the result of an intentiona’ tort, negligence, or abnormally dangerous activity. Choice (B) is incorrect because strict liability for defective products, under section 402A of the Restatement (Second) of Torts, applies only to those engaged in the business of selling the product that is defective. That type of strict liability, therefore, cannot apply to the homeowner, who is merely an owner and user of the mower, not a seller. Note that “abnormaLly dangerous activity” type of strict liability is also not present here. Choice (C) is incorrect because the fact that the homeownerwas using the mower for its intended purpose is not the crucial consideration. For example, if he was negligent, the fact that he was using the mower for its intended purpose would not permit him to avoid liability. He is not liable because no intentional tort, negligence, or basis for strict liability is present, not because he was using the mower for its intended purpose.
- A football player was the star fulllack for the local college football team. After missing two practices, the football player was dropped from the team by the head football coach. Following his dismissal, the football player met with the coach and asked if he could rejoin the team. The coach said that the football player was despised by the other players and under no circumstances could he return to the team. As the football player was leaving the coach’s office, feeling very dejected, the coach then said to him, “Hope you decide to transfer, because everybody hates your guts around here.”
Later that same evening, the football player wrote a suicide note in which he stated, “The coach is responsible for my despondency. If I can’t play football, I don’t want to live.” After swallowing a bottle of Quaalude barbiturates, the football player fell unconscious in his dormitory room. Moments later, the football player’s roommate entered the room and saw his limp body on the floor. The roommate read the suicide note and then attempted to administer aid. Failing to revive him, the roommate picked up the football player and carried him to the college’s first aid center. The football player received prompt medical attention and soon recovered from his drug overdose.
If the football player asserts a claim against the coach based on intentional infliction of emotional distress, the football player will most likely
(A) prevail, because the coach intended to cause him to suffer emotional distress.
(B) prevail, because the coach’s remark did, in fact, cause the football player to suffer emotional distress.
(C) not prevail, because the football player’s drug overdose resulted from his own voluntary act.
(D) not prevail, because the coach acted reasonably under the circumstances, since everyone on the team hated the football player.
- (C) This example illustrates the major problem most students encounter on the Multistate—choosing between two conceivably correct answers. Choices (A) and (B) are incorrect because courts impose liability for intentional infliction of emotional distress only where the defendant’s conduct has been extreme and outrageous. That is a stringent requirement, demanding that the conduct really was beyond all possible bounds of decency so as to be regarded as atrocious. The coach’s conduct was unkind, but not that extreme and outrageous. Choice (D) is incorrect because it speaks to a reasonableness standard, which is a negligence concern. Since the suit is for intentional infliction of emotional distress, negligence is not in an issue. Consequently, the correct answer is choice (C) because the required outrageousness is not present here.
- A stockbroker became despondent because the economy was terrible and all of his clients had lost substantial amounts of money on their investments. The stockbroker decided to end his life by jumping off a bridge. He walked to the middle of the bridge, climbed over the railing, stood on the tiny ledge on the edge of the bridge, and prepared to jump. A taxi driver who happened to be crossing the bridge at that moment saw what the stockbroker was doing. The taxi driver slammed on his brakes, jumped out of his taxi, started running toward the stockbroker, and yelled, “Don’t do it!” Pleased to hear that someone cared about him, the stockbroker changed his mind about committing suicide and decided that he wanted to live. At that moment, however, the stockbroker started to lose his balance, and his feet began to slip off the small, grimy ledge of the bridge. Just as the stockbroker started to fall, the taxi driver reached him and grabbed the back of his jacket. The taxi driver hung on tightly to the jacket and nearly was pulled over the side of the bridge, but he managed to hang on. Using every bit of strength he could muster, the taxi driver pulled the stockbroker back up and onto the bridge. The stockbroker was relieved to have been saved from falling. Unfortunately, the taxi driver suffered a severe injury to his back from the strain of hanging on to the stockbroker and pulling him back up onto the bridge. A statute in the jurisdiction makes attempted suicide a crime.
If the taxi driver asserts a claim against the stockbroker to recover damages for his injuries, will he prevail?
(A) No, because the stockbroker did not intend to harm anyone other than himself.
(B) No, because attempted suicide was a criminal offense in the jurisdiction.
(C) Yes, because the stockbroker put himself in a position of peril.
(D) Yes, because he succeeded in saving the stockbroker’s life.
- (C) Under the “rescue doctrine,” the fact that someone will try to come to the rescue when others are in danger is regarded as a foreseeable occurrence. As Judge Cardozo put it, “danger invites rescue” [Wagner v. International Railway, 232 N.Y. 176, 180 (1921)1. A rescuer who is injured during the rescue attempt, therefore, can recover against the person whose negligence created the need for someone to come to the rescue. This doctrine applies even when the defendant endangers no one’s safety but his own. In this question, the stockbroker put himself in a position of periL and can be held liable for the rescuer’s injuries. Choice (A) is incorrect because it does not matterwhetherthe defendant intended to hurt himself oranyone else. Intent is not necessary. The fact that the plaintiff endangered someone (himself or another person) is enough to create the need for the rescue and to put the rescuer at risk. Choice (B) is incorrect because the rescue doctrine applies regardless of whether the defendant’s conduct is a criminal offense. Choice (D) is incorrect because saving the defendant’s life is not an element required for the plaintiff to assert a claim based on the rescue doctrine.
- On Thursday morning, two plainclothes police officers were shot to death while trying to arrest a bank robber who had just robbed a bank. Following the killings, the police issued an “all-points-bulletin” for the arrest of a 40-year-old Caucasian male (height, 6 feet; weight, 150-155 pounds), who had been seen near the location of the bank robbery.
On Friday, a security guard at a local department store was walking down Main Street when he noticed a tall, African American man who fit the bank robbery suspect’s description in age, height, and weight. The security guard approached the person (and believing him to be the bank robbery suspect), pulled a gun and arrested him. The security guard held the man in custody until the police arrived a short time later. The police officer informed the security guard that he had arrested the wrong person. The man had told the security guard that he was not the man who had committed the bank robbery and shot the police officers. The man was humiliated by the false arrest.
If the man asserts a claim for assault against the security guard, he will
(A) succeed, if the man saw the security guard pointing the gun at him.
(B) succeed, if the security guard’s mistaken belief was unreasonable.
(C) not succeed, because the security guard did not intend to injure the man.
(D) not succeed, because the man did not suffer any injury from the security guard’s act.
- (A) As a general rule, either a Law enforcement officer or a private citizen may arrest without a warrant to prevent a felony or breach of the peace that is being committed—or reasonably appears about to be committed—in his presence. Once the crime has been committed, the private person may still arrest, but his authority depends upon the fact that the person actually did commit the crime, and he must take the full risk of a mistaken arrest. The killing of the police officers did not occur in the security guard’s presence. As a result, he must take the full risk for falsely arresting the man. Therefore, by pointing the pistol at the man, the security guard would be liable for assault. Choice (B) is incorrect because the reasonableness of the security guard’s belief merely goes to the issue of damages and not to whether Liability exists for the tort of assault. Choice (C) is incorrect because even if the security guard had hoped not to have to shoot, he did intend to put the man in apprehension of being injured. Choice (D) is incorrect because injury is not required for assault. It is enough that the plaintiff is placed in fear or apprehension of being injured, as the man was here.
- A man and a brother were identical twins. The man was having an affair with a woman. The woman’s husband was aware of his wife’s relationship with the man. One morning the husband saw the brother, whom he mistakenly believed was the man. The husband walked toward the brother and said, “I’m going to punch you in the face, you rat.” The brother immediately realized that the husband had mistaken him for his brother. The brother had enough time to inform the husband of his mistake but didn’t say anything. When the husband came closer, the brother punched him in the mouth.
If the husband brings suit against the brother for battery, he should
(A) prevail, because the brother did not inform him of his mistake.
(B) prevail, because the husband’s threat was accompanied by an overt act when the husband approached the brother.
(C) not prevail, because the husband was the initial aggressor.
(D) not prevail, because the brother used reasonable force given the threat.
- (A) The Torts questions on the MBE are becoming exceedingly more difficult. In prior years, Torts was a relatively easy subject area. This is no longer the case. Here, choice (D) is an attractive answer because this initially seems to be just a simple self-defense question. To be sure, a person is privileged to use reasonable force to protect himself or herself from unlawful bodily harm or injury. However, this question has a subtle “twist” involving mistake. If a person is about to be attacked, but knows the attack is based on a mistake as to the person’s identify, the person is not privileged to await the attack and then use force to defend himself if he has time to correct the mistake and prevent the attack. In other words, the husband thought he was attacking the man; the brother knew that, and the brother had a chance to correct the husband’s mistaken belief; yet the brother failed to do so. As a result, the brother is not privileged to assert self-defense under these circumstances. Choice (B) is incorrect because the brother does not have the privilege to use self-defense here, regardless of whether the husband’s threat was accompanied by some overt act. Choice (C) is incorrect because the brother had time to inform the husband of his mistake, even though the husband was the initial aggressor. Choice (D) is incorrect because the brother does not have the privilege to use self-defense here because he had time to inform the aggressor of his mistake as to identity and failed to do so.
- A retired cattle rancher was flying his private plane to visit his daughter and grandchildren. While carefully and skillfully operating his airplane, he suddenly realized that one of his engines was gone. He was then forced to make an emergency landing under the reasonable belief that it was necessary to do so for the protection of himself and his airplane. He landed in a farmer’s orange grove and managed to steer the plane and land it between the trees in order to not cause damage to any of them.
If the farmer brings suit against the rancher for trespass, the most likely result is that the farmer will
(A) recover, because the property belonged to a private person.
(B) recover, because the rancher had reasonable grounds to believe that the orange grove belonged to a private person.
(C) not recover, because the rancher was privileged to enter the land of another under the circumstances.
(D) not recover, because the rancher was carefully and skillfully operating his airplane when the engine went out.
- (C) Where a person acts for the protection of himself or his property, he is privileged to enter another’s property to prevent a threatened harm from taking effect. Here, the pilot was privileged to land his plane in the orange grove in order to protect himself and his airplane. Note that if the pilot had caused harm to the property, such as by damaging some of the orange trees, the pilot would be liable for that harm. Choice (A) is incorrect because the fact that the property belonged to a private person does not take away the privilege to trespass to avoid harm. Choice (B) is incorrect because, again, the fact that the orange grove belongs to a private person will not remove the privilege to enter the land to avoid harm. Choice (D) is incorrect because the fact that the pilot was operating the plane skillfully and carefully would not save him from liability for trespass. His privilege to enter the land is based on the necessity of saving himself and his airplane, not the fact that he was flying the plane carefully.
- A hiker was enjoying a journey through a large forest when he was caught in an unexpected blizzard. As it grew dark and the snow continued, he realized he was completely lost. He felt sure that he would freeze to death during the night. When all hope seemed lost, he stumbled across a small cabin. He knocked on the door, but no one answered. He tried to open the door, realized it was not locked, and went inside. In the cabin, he found some food to eat and some blankets that kept him warm through the night. By the next morning, the snow had stopped, and he was able to make his way out of the forest.
If the owner of the cabin sues the hiker, seeking to recover damages for the cost of the food and for the use of his cabin, what is the most likely outcome of the case?
(A) The hiker does not owe anything because the importance of his survival outweighs the property interests of the cabin owner.
(B) The hiker must pay for the cost of the food and the rental value of the cabin for one night.
(C) The hiker must pay for the use of the cabin, but nothing else.
(D) The hiker must pay for the cost of the food, but nothing else.
- (D) The privilege of private necessity is a defense to intentional tort claims. It applies where a person needs to enter or use another person’s property in order to avoid a threatened harm to himself or his own property. For example, if you are about to drown in a flood, you can use a person’s boat to save yourself. While you will not be liable for trespass merely for entering or using the other person’s property, you will be obligated to pay for any actual harm that you cause to it. The hiker acted out of private necessity, to save himself from dying in the blizzard. He, therefore, is not liable merely for entering and using the cabin. He is required, however, to pay for the actual harm caused, so he must pay for the food that he ate. Choices (A), (B), and (C) are incorrect because the hiker will be liable for the cost of the food, but nothing else.
37. A 26-year-old graduate student knew that a classmate was a staunch supporter of animal rights. As they were leaving class one afternoon, the graduate student walked beside the classmate and chanted, “Animal experimentation is great.” The classmate angrily confronted the graduate student and said, “Let’s settle this outside in a fist fight.” The graduate student replied, “You’re on.” The two students went outside and “squared off.” The graduate student threw the first punch and hit the classmate in the face with his bare fist. Unknown to the graduate student, the classmate placed a set of brass knuckles on his fist and hit the graduate student in the face with them. The force of the blow broke the graduate student’s nose. It was later determined that the use of the brass knuckles inflicted exactly the same damage as if the classmate had hit the graduate student with his bare fist. The best argument for rejecting the defense of consent in an action by the graduate student for either assault or battery is that (A) the classmate’s use of the brass knuckles exceeded consent under the circumstances. (B) the classmate was only acting in self-defense. (C) the graduate student’s insults provoked the classmate’s use of excessive force. (D) the classmate’s punch would have inflicted the same harm with or without the use of the brass knuckles.
- (A) Consent is generally a defense to intentional tort claims. Consent is to the defendant’s conduct, rather than to its consequences. Thus, if the plaintiff willingly engages in a boxing match, he does not consent to be killed, although he does consent to the defendant hitting him if he can and, consequently, if death unexpectedly results, his consent to the act will defeat any action for the invasion of his interests. However, a plaintiff who agrees to a fistfight does not consent to being hit with brass knuckles, which is the same invasion by an act of a different character. In sum, a defendant’s privilege is limited to the conduct to which the plaintiff consents. The graduate student’s best argument, therefore, is that the use of brass knuckles exceeded the scope of the consent in this situation. Choice (B) is incorrect because self-defense is not applicable when the issue is consent that has been exceeded. Choice (C) is incorrect because words alone cannot justify exceeding the scope of consent and using excessive force. Choice (D) is incorrect because it was the use of the brass knuckles that is the issue, not the extent of injury that occurred or would have occurred without them.
- A center and a quarterback were 14-year-old eighth graders at the local high school. They were both members of the school’s junior varsity football team. The center weighed 170 pounds and was stocky in build. The quarterback was lanky and weighed about 145 pounds. The center and the quarterback often engaged in friendly tests of strength, such as arm wrestling and weight lifting contests, to see who was stronger.
One afternoon, both youngsters were in the school gymnasium getting ready for football practice. As they were putting on their football uniforms, the center turned to the quarterback and said, “Hey, weakling, why don’t you punch me in the chest with your best shot?” The quarterback replied, “No, I’m afraid I might hurt you.” The center then said, “Are you kidding? You can’t hurt me. C’mon, give me your best shot.” The center stood up, stuck out his muscular chest, and prepared to have the quarterback hit him. The quarterback proceeded to cock his fist and then punched the center in the chest as hard as he could. Immediately thereafter, the center slumped to the floor, gasping for air. Realizing that the center was seriously hurt, the quarterback tried to render assistance but to no avail.
Seconds later the center stopped breathing and died. Unknown to either the center or the quarterback, the center had a defective heart and suffered a heart attack as a result of the blow to the chest.
In a wrongful death action, the quarterback will likely
(A) be held responsible, because he committed a battery by hitting the center in the chest.
(B) be held responsible, because the center’s consent was ineffective.
(C) not be held responsible, because the center’s consent was effective.
(D) not be held responsible, because the quarterback delivered the punch with greater strength than the center anticipated.
- (C) The consent of the person injured will ordinarily bar liability from being imposed for intentional interference with the person or property. Because the center consented to being punched in the chest, the quarterback is not liable for battery. Choice (A) is incorrect because a battery can only exist where the plaintiff does not give permission or consent to be touched. Here, the plaintiff consented to the touching. Choice (B) is incorrect because the center’s consent was effective. Even though the center was 14 years old, a minor may consent. For example, a 14-year-old boy who plays a tackle football game consents to physical contact, and teenagers may consent to engage in fistfights or similar physical encounters. Choice CD) is incorrect because the consent is to the plaintiff’s conduct, not the consequences. In other words, what matters is that the center consented to be punched in the chest, not whether he could anticipate the extent of the harm that would result.
- An owner of an apartment building in a rundown section of town knew that one of his tenants was dealing drugs from his apartment. He also knew that the tenant’s actions had led to some violent confrontations in the past, including one where a gun was discharged in the hallway outside the tenant’s apartment. Because the tenant regularly paid his rent on time, the owner didn’t take any action to try to evict the tenant. One day, a rival drug dealer drove by the tenant’s apartment and fired his gun at the tenant’s front window. One of the bullets passed through the tenant’s living room and went through the wall into the apartment next door. The bullet struck and injured a neighbor who occupied the adjoining apartment.
If the neighbor asserts a claim against the owner to recover damages for his injury, he should
(A) recover, because the owner knew that the tenant was dealing drugs from his apartment.
(B) recover, because the owner was aware that such danger existed because of the tenant’s drug-dealing activities.
(C) not recover, because the owner is not liable for the criminal acts of third persons.
(D) not recover, because the neighbor is outside the zone of foreseeable danger.
- (B) As a general rule, a person is not held liable for failing to control the conduct of a third person unless a special relationship exists between the parties. Relationships that may be treated as special for this purpose include: (1) innkeeper and guest; (2) common carrier and passenger; (3) school and pupil; (4) landlord and tenant; (5) jaiLer and prisoner; and (6) employer and employee. In this regard, a landlord has the responsibility to act for the protection of his tenants and will be held liable if his failure to exercise reasonable care increases the risk of harm to the plaintiff through the criminal ortortious acts of others. As a result, the owner will be subject to liability if he was aware that such a. danger existed on account of his tenant’s drug-dealing activities. Choice (A) is incorrect because it does not provide the best statement of the reason for the special relationship to exist between an owner and his tenants. The fact that drugs were being dealt from the apartment may not alone subject the owner to liability. However, since the owner was aware that dealing drugs from the apartment created a danger, the owner has the responsibility to act for the protection of his tenants. For that reason, Choice (B) is the best answer. Choice (C) is incorrect because a special relationship exists, imposing a duty on the owner to act in preventing harm to the plaintiff through the criminal or tortious conduct ofotherthird persons. Choice (D) is incorrect because if the owner is aware that such danger exists, he has a duty to prevent that danger from harming his tenants.
- A property owner owned a 30-acre tract of land located on the outskirts of a community. After prosperity burst upon the community, the property owner subdivided the property into 30 lots and built townhouses on each of the lots. After selling all 30 of the lots, the various homeowners formed a homeowners’ association. Thereafter, the homeowners’ association adopted a Declaration of Covenants, which provided, inter alia, that each homeowner was required to pay an annual assessment charge to be used “for the promotion of the health, safety, and welfare of residents within the subdivision and for the enhancement of education, social life, and community welfare.”
The homeowners’ association then properly and legally assessed each homeowner an assessment charge of $750. An owner of a townhouse in the subdivision refused to pay the assessment. The townhouse owner was experiencing financial problems and claimed that she couldn’t afford to pay the $750. As a result, the president of the homeowners’ association sent the townhouse owner the following letter:
“Article 1 of the Declaration of Covenants requires all homeowners to pay their annual assessment charge on the first day of January for the said year. Because of your failure to make this payment, the homeowners’ association shall have the right to collect the amount due by action of law. Your prompt attention to this matter is greatly appreciated.”
Three weeks elapsed, and the president of the homeowners’ association did not receive any response from the townhouse owner. He then wrote her a second letter that stated:
“Before it institutes legal action, this is the homeowners’ association’s final demand for your $750 assessment charge. It is unfortunate that deadbeats like you ruin our community.”
A copy of this letter was mailed to the other
29 residents of the development. The president
even sent a copy of the letter to the editor of the
county newspaper, which had a circulation of
20,000 subscribers. In its next edition, the county
newspaper published the president’s letter on its
editorial page.
If the townhouse owner sues the president for defamation, she will probably
(A) not prevail, because the president was acting to protect a legitimate public interest.
(B) not prevail, because the statement involves only an expression of opinion.
(C) prevail, because the president obviously acted with malice by sending the letter to the newspaper.
(D) prevail, because the language of the president’s letter would be highly offensive to reasonable persons.
- (B) The tort of defamation requires the plaintiff to prove that the defendant intentionally communicated (i.e., published) a defamatory matterto some third person, who reasonably understands that the plaintiff’s reputation is lowered in the estimation of at least a substantial minority of the community, or such that third persons are deterred from associatingwith the person to whom the matter refers. Therefore, only statements of fact can be actionable as being defamatory. The townhouse owner is suing the president for calling her a “deadbeat.” She will not prevail, because this is simply a statement of the president’s opinion of the townhouse owner. Choice (A) is incorrect because even though the common law provides a conditional privilege for fair comment on matters of general public interest, the president’s letter regarding failure of one single homeowner to pay annual maintenance dues for her townhouse does not involve a matter of such public concern as to reasonably permit publication to 20,000 newspaper subscribers. Choice (C) is incorrect for failing to address the more fundamental requirement that only statements of fact are actionable, regardless of malice. Choice (D) is incorrect because even if the president’s letterwas highly offensive to reasonable persons, it would not be defamatory, since it was merely a statement of opinion.
- A manufacturer of snowmobiles produced one model, Model IA, which was a very popular snowmobile. On each of the Model 1 A snowmobiles produced, the manufacturer installed a speed-regulating device that prevented the snowmobiles from exceeding 50 m.p.h. The manufacturer distributed the snowmobile through wholesalers and retailers across the country. In its sales brochures and advertising materials, the manufacturer advises all of its distributors against making any modifications on snowmobiles without first conferring with the manufacturer’s staff of engineers.
The operator of a shop in a mountain community had the exclusive right to distribute Model 1A snowmobiles in the area. Amid complaints from previous buyers regarding Model lA’s speed limitations, the operator removed the speed regulators from all the Model 1A snowmobiles in stock.
A buyer purchased from the operator one of the Model 1A snowmobiles that had the speed regulator removed. One day, the buyer was driving the Model 1A snowmobile through a snow-covered field, at speeds in excess of 70 m.p.h. The snowmobile hit a snow-covered rock, causing the buyer to lose control of the snowmobile, crash into a tree, and suffer serious injuries.
If the buyer institutes a strict liability in tort action, he will most likely recover against
(A) the manufacturer only.
(B) the manufacturer, even though it was unaware of the removal of the speed regulator.
(C) the operator only.
(D) both the manufacturer and the operator.
- (C) Under Section 402A of the Restatement (Second) of Torts, a defendant engaged in the business of selling a product may be held strictly liable for harm caused by the product being in a defective and unreasonably dangerous condition. This strict liability can apply to a seller at any level of the chain of distribution of the product. In other words, it can be imposed on a manufacturer, a wholesale distributor, and a retail dealer. A defendant will only be strictly liable, however, if the defect existed at the time the defendant sold the product. For example, a manufacturer will be strictly liable only for defects that existed when the product left its hands and not for defects created later by a downstream seller altering or mishandling the product. In the present case, the alleged defect is the lack of speed regulators. That defect existed when the operator sold the product, but not when the manufacturer sold the product, so only the operator can be held strictly liable. Choice (A) is, therefore, incorrect. Choice (B) is incorrect because regardless of whether the manufacturer knew about the regulators being removed, the product was not defective when sold by the manufacturer and, therefore, the manufacturer is not strictly liable. Note that another cause of action, such as negligence, might succeed if the alteration of the product was foreseeable and a manufacturer exercising reasonable care would have taken steps to prevent the alteration from occurring. This question, however, refers only to strict liability in tort. Choice (D) is incorrect because the manufacturer would not be liable because the snowmobile was not defective until the operator removed the speed regulators.
- A buyer purchased a shovel at a local hardware store. The shovel had a metal blade and a wooden handle. While the buyer was using the shovel to dig out a tree stump in his back yard, the handle broke in half, causing the buyer to fall and injure his back.
The buyer brought a negligence claim against the manufacturer of the shovel. He proved that the shovel broke because of a manufacturing defect that made the handle too weak and that the manufacturer acted unreasonably in allowing this to happen. While conceding that the handle had a defect that caused it to break, the manufacturer established that if the buyer had been using the shovel carefully, he would not have fallen down and would not have been injured when the shovel broke.
Assume that the case is governed by the law of a jurisdiction that follows traditional contributory negligence rules.
Who is most likely to prevail?
(A) The buyer, because the manufacturer conceded that the shovel was defective.
(B) The buyer, because the manufactuter failed to exercise reasonable care in the production and inspection of its product.
(C) The manufacturer, because the manufacturer did not guarantee that the shovel would never break.
(D) The manufacturer, because the buyer’s negligence in the way he used the shovel was one of the causes of his injury.
- (D) Traditionally, contributory negligence was a complete bar to a negligence claim. In other words, if a plaintiffs injury was caused both by a defendant’s negligence and also by the plaintiff’s own failure to act reasonably, the plaintiff would not be able to recover any damages. However, while contributory negligence was a defense to a negligence claim, it was not a defense to a strict liability in tort claim against the manufacturer or seller of a defective product. The traditional rule of contributory negligence has been replaced in most states by comparative fault, under which the jury has the task of allocating percentages of fault to each party. Comparative fault means that a plaintiff will be able to recover despite being negligent, although the plaintiff’s negligence may reduce the damages awarded. Comparative fault is a defense to negligence claims and strict liability claims concerning defective products. On the Multistate Bar Exam, you are instructed to assume that comparative fault applies unless the question states otherwise. This question specifies that traditional contributory negligence rules apply. Under those rules, contributory negligence would be a complete defense to a negligence claim. This question says that the buyer is bringing a negligence claim, and so the manufacturerwill prevail if there was contributory negligence. Choice (A) is incorrect because the buyer is suing for negligence. If he were bringing a strict liability in tort claim instead of a negligence claim, then choice (A) would be correct. Choice (B) is incorrect because the buyer’s negligence claim is barred by the defense of contributory negligence. Choice (C) is incorrect because it does not matter whether the manufacturer guaranteed that the shovel would not break. The manufacturer could be liable for negligence or selling a defective product even if it never gave any guarantees.
- After winning a big antitrust case, an attorney and a few associates decided to celebrate and have a few drinks at a popular downtown watering hole. After having two gimlets (a cocktail containing vodka and lime juice), the attorney left his friends and drove home.
The attorney, who was a bit tipsy, began driving in an erratic and reckless manner. He was traveling at an excessive speed through a residential section of town when he approached a sharp curve in the roadway. Trying to negotiate the turn, the attorney lost control of his vehicle and veered off the road, landing on the front lawn of a woman’s property.
If the woman asserts a claim against the attorney for intentional trespass, she will most likely
(A) prevail, because the attorney was operating his car recklessly.
(B) prevail, because the attorney entered onto her property.
(C) not prevail, because the attorney did not damage her land.
(D) not prevail, because the attorney did not intentionally enter onto her property.
- (D) A person is subject to liability for intentional trespass if he intentionally enters land possessed by another person, regardless of whether he causes any actual harm to the Land. Based upon the given facts, the attorney did not intentionally drive his vehicle onto the woman’s property. Rather, he Lost control of his vehicle whUe trying to negotiate a sharp curve in the highway. The woman could try to sue for negligence because the attorney was operating his car in a reckless manner, but the attorney would be liable for negligence only if he caused some damage to the land or other actual harm. The woman, therefore, cannot prevail on an intentional trespass claim without proof of intent, so the woman will not be able to hold the attorney liable for this incident. Choice (A) is incorrect because the fact that the attorney was driving recklessly is not sufficient to generate liability for intentional trespass. Note that the attorney would be treated as having intentionally entered the woman’s property if the attorney had acted with knowledge that he was substantially certain to wind up on the woman’s property. Although the attorney was reckless, he did not have the knowledge of a substantial certainty that would make him liable for an intentional trespass. Choice (B) is incorrect because even though the attorney entered the woman’s property, he did not do so intentionally. Choice (C) is incorrect because damages are not a required element of an intentional trespass.
- A man owned a beautiful tract of land in a mountain resort. The property was purchased by the man and used as a family vacation retreat. About 200 yards of the man’s property bordered along a lake shoreline. A fisherman lived on a stream that flowed along one boundary of the man’s land and ran into the lake. When the man acquired ownership of the property, he had a channel dredged across his land from the stream to the lake at a point some distance from the mouth of the stream. The fisherman erroneously believed that the channel was a public waterway. Because the channel served as a convenient shortcut to the lake, the fisherman made frequent trips across the channel in his fishing boat. In no way did the fisherman’s use of the channel cause any harm or damage to the man’s property.
After the man learned about the fisherman’s use of the channel, he requested that the fisherman desist further entry onto the waterway. The fisherman, who until that time was unaware of the man’s ownership claim, agreed. Nonetheless, the man brought suit against the fisherman to recover damages for trespass.
Judgment is likely to be for whom?
(A) The fisherman, because when he used the channel he believed that it was a public waterway.
(B) The fisherman, because he caused no damage to the man’s land.
(C) The man, but recovery is limited to nominal damages for the fisherman’s intentional use of the channel.
(D) The man, because the fisherman should have known that the waterway was not open for public use.
- (C) Mistake is no defense to a claim for the intentional tort of trespass to land. A person is liable for trespass if he enters another person’s land because of mistaken belief that it is his own, even if the mistake is completely understandable and reasonable. In other words, trespass requires only that the defendant intended to enter the piece of land in question, not that the defendant intended to enter a place that the defendant believed was someone else’s land. Here, the fisherman erroneously believed the channel was a public waterway when, in fact, it was owned by the man. Even if this was a reasonable mistake made in good faith, the fisherman is still liable fortrespass, although the damages will be nominal because no real harm was caused. Choice (A) is incorrect because a mistaken understanding about who owned the property in question is no defense for an intentional trespass. Choice (B) is incorrect because damages is not a required element of an intentional trespass. Choice (D) is incorrect because the fisherman’s knowledge is irrelevant to his liability for trespass. As stated above, mistake is no defense to an intentional trespass, so the fisherman is liable even if he should not have known that the waterway was not open to the public.
- A pharmacist was employed by a drug store. A long-time customer of the drug store presented a prescription to the pharmacist for the medication Clinoril, which was prescribed by her physician to treat her arthritic condition. The pharmacist was in a rush that day because of a backlog of prescriptions to fill, and he misread the customer’s prescription. He filled her prescription with Clinitest, a caustic substance not for internal use. He labeled the container with the instructions for Clinoril: “Take one tablet twice a day.”
The customer followed the instructions, consumed the Clinitest, and suffered severe injuries to her stomach and esophagus.
If the customer brings a strict liability in tort action against the pharmacist, what is his best defense?
(A) It was the cashier and not he who personally received the money for the medication.
(B) He was not a seller of the product upon whom strict liability may be imposed.
(C) He exercised reasonable care under the circumstances.
(D) The drug store was not the manufacturer upon whom ultimate liability falls.
- (B) Section 402A of the Restatement (Second) of Torts provides for strict liability to be imposed on a person who sells a defective product. It can apply to a seller at any stage of the chain of distribution of the product, such as the manufacturer, wholesale distributor, or retail dealer. It only applies, however, to someone who is engaged in the business of selling the product. For example, while strict liability would be imposed on the company that manufactured and sold a product, it would not be imposed on an employee of the manufacturing company who merely worked in the factory and put together the product. Likewise, a retail store can be held strictly liable for selling a defective product, but strict liability would not be imposed on the employees of the store. In this question, the pharmacist’s best defense would be that he was merely an employee of a company that sold the product, but was not himself the seller. Choice (A) is incorrect because the fact that the pharmacist did not personally receive the money from the customer is not crucial. The cashier did receive the money, but the cashier is also just an employee and is not subject to strict liability. Choice (C) is incorrect because whether the defendant exercised reasonable care is irrelevant in a strict liability action. It is significant for a negligence claim, but not for strict liability. Choice (D) is incorrect because other sellers, as well as manufacturers, are held strictly liable for defective products that cause harm.
- A woman had her kitchen renovated. She purchased a new dishwasher, and the contractor doing the renovations installed it in the woman’s kitchen.
Two months later, the woman was entertaining a friend at her home. As they talked, they noticed that the dishwasher was operating strangely, repeatedly stopping and starting. They saw sparks start flying out from underneath the dishwasher. The friend said, “It must have a short circuit or something. Don’t touch it or you might get electrocuted.” The woman asked if she should go down to the fuse box in her basement to cut off the electricity. The friend decided that he would take a look inside the dishwasher first. When he touched the metal handle on the dishwasher’s door, he received a violent electric shock. The dishwasher had an internal wiring defect that caused it to malfunction. The contractor who installed the dishwasher failed to adequately ground the machine; if he had done so, the electric current would have been led harmlessly away. The machine carried instructions for proper grounding, but the contractor did not follow them.
All of the relevant events occurred in a jurisdiction that follows traditional contributory negligence and assumption of risk rules.
If the friend brings a strict tort liability claim against the manufacturer of the dishwasher for his injuries, the probable result is that the friend will
(A) recover, because the dishwasher was defectively made.
(B) recover, because the manufacturer is vicariously liable for the improper installation.
(C) not recover, because the friend assumed the risk by trying to open the dishwasher.
(D) not recover, because the friend was not the purchaser of the dishwasher.
- (C) Implied assumption of risk traditionally was a defense to a negligence claim or a strict Liability claim concerning a defective product. Implied assumption of risk meant that the plaintiff voluntarily chose to encounter a known risk. For example, if a person knew that his car had defective brakes but decided to drive the car anyway, the person would be barred from holding liable the manufacturer or auto dealer that sold the car. In most states, implied assumption of risk is now simply handled as part of comparative fault. In other words, if a plaintiff voluntarily chose to encounter a known danger, that is not necessarily a total bar to liability but, instead, it is simply a reason the jury may choose to assign some fault to the plaintiff when it decides each party’s share of the comparative fault. Here, the friend knew the dishwasher was malfunctioning and even saw sparks flying out of the dishwasher. He knew there was a risk of receiving an electric shock, but he chose to touch the dishwasher anyway. The question says that traditional assumption of risk rules appLy and, therefore, the friend’s voluntary choice to encounter the known risk of electric shock completely bars him from holding the manufacturer liable. Note that if the question did not specify that traditional rules applied, you would need to assume that pure comparative fault applies, and the friend’s decision to touch the dishwasher would be dealt with as part of the comparative fault allocation, and it would be likely to reduce the friend’s recovery by some percentage, but not bar liability completely. Choice (A) is incorrect because implied assumption of risk would bar the friend’s claim. Choice (B) is incorrect because there is no reason the manufacturer would be vicariously liable for the improper installation of the product and, in any event, implied assumption of risk would bar the claim. Choice (D) is incorrect because an injured person can bring products liability claims even if he was not the purchaser or consumer of the product. No “privity of contract” is required, and even a random bystander who happens to be injured by the product can sue and recover damages.
- A buyer purchased a gallon of nonfat milk from a market. The milk, which was sold in a plastic container, had been processed and packaged by a dairy. That evening, the buyer was preparing dinner and poured himself a glass of milk from the container purchased at the market. He then sat down to eat his meal and proceeded to take a mouthful of milk. As he did so, the buyer felt something furry lodge in his mouth. He immediately spat out the furry object and saw that it was a dead mouse. The buyer suffered severe emotional distress but did not suffer any physical illness or injury.
If the buyer asserts a claim against the dairy based on negligent manufacturing, he will most likely
(A) recover, because he suffered severe emotional distress.
(B) recover, under the doctrine of res ipsa loquitur.
(C) not recover, because the dairy’s negligence only caused mental disturbance.
(D) not recover, because the buyer’s proper cause of action is for intentional infliction of mental distress.
- (C) Based on the holding in Sullivan v. H. RHood& Sons, 168 N.E.2d 80 (Mass. 1960), no recovery will be allowed for mental disturbance alone unaccompanied byphysical injury upon finding a dead mouse in a milk bottle. Where the defendant’s negligence causes only mental disturbance, without accompanying physical injury, illness, or other physical consequences, and in the absence of some independent basis for tort liability, the great majority of courts still hold that there can be no recovery. The rationale for denying such recovery is based upon the notion that such mental harm is often temporary and relatively trivial. Because claims may easily be falsified or exaggerated, merely negligent, rather than intentional, conduct should not be the basis for imposing a disproportionate financial burden on a defendant. Choice (C) is thus correct. Choice (A) is incorrect because severe emotional distress is not compensable under negligence principles in these circumstances. Choice (B) is incorrect because res ipso loquitur merely helps a plaintiff to prove that there was negligence, but does not overcome the lack of physical injury that bars the plaintiff from recovering here. Choice (D) is incorrect because there is no indication that the milk company intentionally put a mouse in the bottle, so liability for intentional infliction of emotional distress could not be imposed.
- On Friday night, a driver was injured in an automobile accident after his vehicle was sideswiped by an unidentified motorist. Following the accident, the driver was admitted to the hospital where he was treated for his injuries. While at the hospital, the driver was diagnosed with a broken back. He was put in a body cast and then placed in traction. The driver’s physician assembled the traction apparatus and positioned the driver’s left leg in an overhead stirrup at a 40-degree angle. The driver was instructed to lie on his back on the bed.
On Saturday morning after breakfast, the driver requested a bedpan from a nurse. She was placing the bedpan under the driver’s body when the stirrup, which was holding his leg, broke. This caused the driver’s leg to fall against the metal framing along the side of the bed, fracturing his tibia. The traction apparatus was defective because the manufacturer had not included a safety latch on the stirrup device to prevent its falling off if not securely fastened in place.
If the driver asserts a claim against the manufacturer, he will most likely
(A) prevail, because the stirrup broke while the traction apparatus was in normal use.
(B) prevail, because the manufacturer’s failure to include a safety latch made the traction apparatus defective.
(C) not prevail, because the traction apparatus was no longer within the control of the manufacturer.
(D) not prevail, if the physician did not properly attach the stirrup when assembling the traction apparatus.
- (B) Often in torts questions, the cause of action is not designated, and the question simply reads: “If A asserts a claim against B, judgment for whom?” As a consequence, you must first determine the proper cause of action. Here, the plaintiff is suing the manufacturer of the traction device. In accordance with Section 402A of the Restatement (Second) of Torts, a manufacturer or seLLer of goods will be strictly liable for injuries caused by a defective product. Thus, since the traction apparatus was defective because of its failure to include a safety latch, the driver will prevail. Choice (B) is, therefore, the best answer. Choice (A) is incorrect because the mere fact that the apparatus broke during normal use does not by itself make it defective, nor does it prove that the manufacturer was negligent. Choice (C) is incorrect because a seller of a defective product remains strictly liable even after it leaves his control. Choice (D) is incorrect because a negligent act of a third party (like the physician) does not necessarily bar the manufacturer from being held liable. The physician’s negligence would block the manufacturer from being held liable only if the physician’s conduct was such an unforeseeable and significant cause of the accident that it must be treated as a supervening cause that breaks the chain of proximate causation between the manufacturer and the injured plaintiff. The facts do not indicate that the physician’s actions were unforeseeable, so choice (0) is not the best answer here.
- A woman broke her leg while skiing. She was taken to a hospital where she was treated by a doctor who determined that the woman would need surgery to set the broken bones so that they would heal properly. During the surgery, the doctor inserted several metal screws into the woman’s leg to hold the bones together. Although the hospital charged the woman a substantial fee for the doctor’s services, the bill did not include a separate charge for the cost of the bone screws.
A few months after the surgery, the woman’s leg still was not fully healed. The doctor eventually concluded that the screws that were inserted into the woman’s leg had been defective. The woman was forced to have a second surgery to replace the defective screws.
If the woman asserts a strict tort liability claim against the doctor and the hospital for using defective screws in her surgery, she will most likely
(A) prevail, because the doctor and the hospital provided a product to her that was defective and unreasonably dangerous.
(B) prevail, because the doctor and the hospital probably would have realized the screws were defective if they had carefully examined them.
(C) not prevail, because the doctor and the hospital did not manufacture the screws and, therefore, the defect in the screws was not their fault.
(D) not prevail, because the doctor and the hospital will be treated as service providers rather than product sellers.
- (D) Under Section 402A of the Restatement (Second) of Torts, a person can be held strictly liable for selling a product that is defective and unreasonably dangerous. This form of strict liability applies only to the sale of products, and not to the provision of services. Where some type of product or device (such as a pacemaker, prosthetic item, or artificial joint) is implanted into a patient during surgery, most courts hold that the doctor and hospital should be treated as providing health care services and not as a selLer of a product. That means the patient may bring a strict liability claim against the manufacturer of the item, but the patient will need to sue the doctor or hospital for negligence (i.e., medical malpractice) rather than strict liability. Here, the patient’s strict liability claim against the doctor and hospital will fail because the defendants will be treated as providing health care services, rather than selling a defective product. Choice (A) is wrong because the doctor and hospital will not be treated as having sold the bone screws, and so strict liability will not apply. Choice (B) is incorrect because the question asks about strict liability, and whether the doctor and hospital should have realized the screws were flawed through a careful inspection is relevant to a negligence claim but not strict liability. Choice (C) is wrong because, in general, all sellers of a defective product, not just the manufacturer, will be subject to strict liability. Strict liability does not require proof that anyone was at fault, so the fact that the doctor and hospital were not at fault is irrelevant here.
- A consumer purchased a new station wagon from a car dealer. The car dealer performed a thorough inspection of the station wagon before the sale was completed. One evening, the consumer was driving along a city street when he saw the traffic light facing him turn from green to amber. He sped up, hoping to cross the intersection before the light turned red. However, he quickly realized that he could not do so and applied the brakes, which failed because of a defect in the brake mechanism. The consumer then swerved to avoid hitting a bus that was crossing the intersection at a right angle to him. As a result of the swerve, the consumer’s car rode up on the sidewalk and overturned, pinning a pedestrian. Both the consumer and the pedestrian were severely injured.
If the consumer asserts a claim against the car dealer based on strict liability in tort, will the plaintiff prevail?
(A) Yes, because the brakes failed while the consumer was driving his car.
(B) Yes, because the brakes failed because of a defect present when the consumer purchased the car.
(C) No, because the consumer contributed to his own injury by speeding up.
(D) No, because the car dealer carefully inspected the car before selling it.
- (B) Under Section 402A of the Restatement (Second) of Torts, a person engaged in the business of selling a product can be held strictly liable if the product causes harm because it is defective and unreasonably dangerous. Thus, if the car was sold with defective brakes, the car dealer will be strictly liable for the consumer’s injuries. ConsequentLy, choice (B) is the best answer. Choice (A) is incorrect because the mere fact that an accident occurs or a product fails to work properly is not enough to make strict liability apply. The plaintiff must show that the product had a defect at the time it was sold. Choice (C) is incorrect because under pure comparative fault principles (which apply on the Multistate, unless a question indicates otherwise), the fact that the consumer’s own negligence contributed to his injury would resuLt in the jury reducing his damages by some percentage but would not bar his claim completely. Note that if the question indicated that traditional contributory negligence rules apply, choice (C) would still be wrong because contributory negligence was not a defense to strict liability claims. Choice (D) is incorrect because strict Liability means the seller of the car would be liable if it was defective at the time of sale, even if the seller exercised reasonable care to inspect it.
- A farmer purchased a new pickup truck from a local automobile dealership. A few weeks later, the farmer was using the truck to haul a load of supplies to his farm. As he drove down a small hill toward an intersection with a stop sign, he applied the brakes, but they failed to work, and the truck did not slow down. The farmer could not stop for the stop sign. He saw a pedestrian crossing the street at the intersection and tried to steer around him, but was unable to do so. The pickup truck struck the pedestrian and seriously injured him.
If the injured pedestrian asserts a negligence claim against the auto dealership that sold the truck to the farmer, and if it is conclusively proven that the brake failure resulted from a manufacturing defect in the car, will the pedestrian prevail?
(A) No, because the pedestrian was not a purchaser or user of the truck.
(B) No, if the farmer was negligent in being unable to steer around and avoid hitting the pedestrian.
(C) Yes, because the auto dealer placed a defective car into the stream of commerce.
(D) Yes, if the defect could have been discovered through the exercise of reasonable care by the auto dealer.
- (D) The seller of a product can be held liable for negligence if he fails to exercise reasonable care, and for strict liability if he sells a defective product. Obviously, a seller’s negligence may take a number of forms. Most frequently, it consists merely in failing to exercise reasonable care to inspect the goods to discover defects, or in preparing them for sale. Therefore, the auto dealer can be held liable for negligence if it failed to discover a defect in the truck that would have been detected by a seller exercising reasonable care. Choice (A) is incorrect because the products liability claims (negligence and strict liability) can be brought by people other than the purchaser or user of the product. Even a mere bystander who happens to be injured by the product can bring these claims. Choice (B) is incorrect because even if the farmer did a poor job of steering the truck after the brakes failed, that negligence by the farmer would not prevent the auto dealer from being held liable for its own negligence that was also a cause of the accident and injury. Choice (C) is incorrect because this question asks about a negligence cause of action rather than strict liability.
- A scientist used his car to transport a large quantity of highly flammable petroleum derivatives that he needed for his scientific research. The petroleum derivatives were sold in ordinary glass gallon jugs.
Shortly after putting the jugs in the back of his car, the scientist was driving along a city street. He was thinking about a difficult scientific question and not paying attention to his driving. As a result, he lost control of the car and drove up onto the sidewalk. The car flipped over. The glass jugs in the back of the car were broken, and the chemicals in them spilled out onto the sidewalk.
Moments later, a doctor who witnessed the accident came running over to render medical assistance. As he approached the overturned car, however, the doctor slipped on the petroleum derivatives that had spilled onto the sidewalk. The doctor fell and fractured his ankle.
If the doctor asserts a claim against the scientist based on strict liability, will the doctor prevail?
(A) Yes, because the scientist was engaged in an abnormally dangerous activity by transporting highly flammable petroleum derivatives in his car.
(B) Yes, because the transportation of flammable petroleum derivatives in glass jugs necessarily involves a high degree of risk of serious harm.
(C) No, because the doctor assumed the risk by voluntarily acting as a Good Samaritan.
(D) No, because it was unforeseeable that the doctor’s injury would result from the type of harm threatened.
- (D) Usually, where a type of conduct is subject to strict liability, it is obvious what sort of harm is expected to result from the conduct. For example, the keeping of vicious animals invoLves the risk that human beings or other animals will be attacked; the risk of abnormally dangerous things and activities, such as high tension electricity or blasting, is obvious. However, in some situations, conduct that is subject to strict liability will result in an unexpected type of harm. In these circumstances, strict liability will not be imposed if the harm that occurs is not within the scope of the risk that makes the conduct subject to strict liability. Here, transporting highly flammable chemicals in one’s car is an abnormally dangerous or ultrahazardous activity because it is so likely to result in a fire or explosion. The possibility of someone slipping on the chemicals is not the risk that makes that activity abnormally dangerous, and so strict liability will not apply. Choice (A) is incorrect because even though the transport of highly flammable chemicals is an abnormally dangerous activity, someone slipping and falling on the chemicals is not the sort of risk that would be reasonably foreseen and that makes that activity abnormally dangerous. Choice (B) is incorrect because even though the transportation of the flammable petroleum in glass jugs is very dangerous, the plaintiffs slip and fall are not the foreseeable consequences of that dangerous activity. Choice (C) is incorrect because assumption of risk means that one voluntarily encounters a known risk. Although the doctor may have assumed certain risks in going to help at the accident scene, such as the risk of a possible explosion or fire, he did not know there was a risk of falling because of a slippery substance on the sidewalk.
- A company manufactures a popular dandruff shampoo. On the shampoo’s box cover is a warning label advising consumers to make a “patch test” before applying. The label states that a “patch test” is necessary because a small percentage of the population may be allergic to the chemicals contained in the product and susceptible to suffering scalp irritation.
A Russian immigrant recently moved to the United States from Moscow. The immigrant cannot read or speak English. One day, the immigrant purchased a bottle of the shampoo from a drug store. She looked at the label warning but did not understand it. After throwing away the box cover, she used the shampoo without making a “patch test.” Minutes later, she began experiencing an allergic reaction and scalp irritation. This was followed by hair loss attributed to the shampoo.
If the immigrant brings suit against the company for strict products liability, she will most likely
(A) win, because she suffered injury from her use of the product.
(B) win, because the manufacturer was aware that a small percentage of the population would suffer an allergic reaction to the shampoo.
(C) lose, because she didn’t read or speak English. (D) lose, because she assumed the risk by not making the “patch test.”
- (C) A product may be unreasonably dangerous if there is an inadequate warning regarding its use. The warning on the shampoo bottle was adequate. American manufacturers are not required to place warnings in different languages on products sold within the United States. The plaintiff, therefore, will lose because the injury was attributable to her inability to read English, not to any flaw in the product or its warnings. Choice (A) is incorrect because even though the plaintiff suffered injury from using the product, the manufacturer’s warning was adequate; therefore, no liability can be imposed. Choice (B) is incorrect because while manufacturers are obligated to warn about the presence of ingredients to which a substantial number of people may be allergic, the manufacturer in this instance provided an adequate warning on the bottle. Choice (D) is incorrect because assumption of risk exists where the plaintiff is subjectively aware of a risk and chooses to encounter it anyway. In other words, the plaintiff must actually know about the danger. Here the plaintiff could not read the warning, so she was not aware of the risk.
- A woman was driving home after attending a concert. While talking on her cellphone, she momentarily took her eyes off the road. The woman then drove through a red light and struck a police car driven by a police officer, injuring him. At the time of the accident, the police officer was returning from an emergency dispatch.
The police officer sued the woman to recover damages suffered in the auto accident. At trial, the police officer presented evidence that he was returning from an emergency scene when the woman drove through a red light and struck his patrol car. At the conclusion of the plaintiff’s presentation of evidence, the woman moved for a summary judgment claiming the “firefighter’s rule” barred recovery.
The motion for summary judgment should be
(A) granted, because the accident would not have occurred but for the emergency.
(B) granted, because the police officer assumed the risk of injury by answering the emergency call.
(C) denied, because the police officer’s injury was not related to any special danger of his job.
(D) denied, because the firefighter’s rule does not apply to police officers.
- (C) The “firefighter’s rule” or “professional rescuer’s rule” provides that a professional rescuer (such as a firefighter or police officer) generally cannot hold someone liable for neglige.nce that creates a need for the rescuer’s services. For example, if you negligently start afire, firefighters come to put it out, and if one of them is injured while doing so, you will not be liable for the firefighter’s injuries. The policy rationales for this rule include: (1) the fact that we all pay taxes to cover the costs of services such as firefighting and police; (2) professional rescuers will be entitled to workers’ compensation benefits for on-the-job injuries; and (3) we do not want fear of liability to discourage people from caLling for help when they need it. This rule only applies, however, to risks that are inherent in the professional rescuer’s job and that are the very reason forthe rescuer’s presence at the scene. In other words, if you negligently start a fire in your house, you will not be liable for injuries that a firefighter suffers trying to put out the fire. However, if you happen to drive past a house where firefighters are working to extinguish a fire and you carelessly run over one of the firefighters, you will be liable. In this question, the woman did not do something negligent that created an emergency requiring the assistance of police officers. Instead, she simply ran a red light and hit a police officerwho happened to be driving through the intersection at that moment. This sort of accident is not a risk inherent in police work, and the woman’s negligence was not the reason the police officer was at that location. As such, the woman’s motion for summary judgment will be denied. Choice (A) is incorrect because the firefighter’s rule does not apply here, even though the police officer was returning from an emergency call, because that emergency call was completely unrelated to the plaintiff. The rule would apply if the plaintiff negligently created an emergency situation and then the police officer was injured while responding to it. Choice (B) is incorrect because the police officer did not assume the risk of being hit by a driver who had driven through a red light. The police officer was simply returning from the emergency calL at the time and had no reason to suspect such an occurrence might happen. Choice (D) is incorrect because police officers and other professional rescuers are covered under this rule, not just firefighters.
55. A first-class passenger on an airplane flight from Atlanta to Chicago was seated next to a middle-aged salesman who was returning home after attending a business convention in Atlanta. The passenger, who was extremely exhausted after a long and hectic day, tried to sleep during the flight but was constantly being annoyed by the salesman. Once the flight departed, the salesman started ordering drinks of Scotch and water from the flight attendant, and became inebriated. When the passenger refused to talk to him, the salesman became very abusive to the passenger. Although there were many empty seats available on the plane in the first-class section, the passenger decided to remain seated next to the salesman. Finally, after the flight attendant had served the saleman his tenth drink of Scotch, he became belligerent and punched the passenger in the mouth. The passenger’s two front teeth were knocked loose and she suffered a cut lip. If the passenger asserts a claim against the airline based on negligence, she will most likely (A) prevail, because, as a first-class passenger, she was owed a special duty of care by the airline. (B) prevail, because the flight attendant should have been aware that her conduct caused an unjustifiable risk of harm. (C) not prevail, because the airline is not vicariously liable for the tortious conduct of its passengers. (D) not prevail, because the passenger assumed the risk by not moving to another seat away from the salesman.
- (B) A person generally does not have a duty to protect others from being harmed by third parties. For example, if you see someone being attacked by a criminal, you might choose to help the victim, but you generally do not have a legal obligation to do so. A legal duty does exist, however, if certain “special relationships” are involved. You might have a special relationship to the crime victim that obligates you to protect him, oryou might have a special relationship to the criminal that obligates you to control him. One of the special relationships traditionally recognized is the relationship between a common carrier (such as a railroad or airline) and its passengers. The common carrier has a duty to its passengers to take reasonable actions to protect them against unreasonable risk of physical harm. This duty to protect extends not only to risks arising out of the common carrier’s own conduct, but also to risks arising from acts of third persons, whether they be innocent, negligent, intentional, or even criminal. The flight attendant’s failure to stop serving drinks to the salesman in light of his inebriated state and abusive behavior posed an unreasonable risk of harm to the passenger. The airline will be vicariously liable for the flight attendant’s negligent conduct under the doctrine of respondeat superior. The passenger will, therefore, prevail. Choice (A) is incorrect because the airline has no greater duty to first-class passengers than to other passengers. Choice (C) is incorrect because even though the airline is notvicariously liable forthetortious conduct of its passengers, it is vicariously liable for the tortious conduct of its employees, and thus the airline can be liable for the negligent conduct of the flight attendant in serving alcohol to an already intoxicated passenger. Choice (D) is incorrect because in order for assumption of risk to apply, the plaintiff must be actually (i.e., subjectively) aware of the risk. Here, the plaintiff did not know that the salesman would strike her.
- A customer went into a bar to have a drink. The bar was crowded because a championship boxing match was being shown on the television sets in the bar, and many people had gone to the bar to watch it. During the second round of the boxing match, someone bumped into the customer, causing him to spill a drink on his pants. “Hey, watch what you’re doing, jerk!” the customer said. The person who had bumped the customer was an employee of the bar hired as a bouncer to deal with unruly customers. Angered by the customer’s statement, the employee immediately turned around and punched the customer in the mouth.
If the customer asserts a battery claim against the owner of the bar, the customer will most likely
(A) prevail, because it was reasonably foreseeable that fights would occur in a crowd of people who were drinking a lot of alcohol.
(B) prevail, because the person who punched the customer was employed as a bouncer.
(C) not prevail, because the owner did not touch the customer.
(D) not prevail, because the owner should not be responsible for the intentional acts of the employee.
- (B) Under the doctrine of respondeat superior, an employer can be held vicariously liable for an employee’s tortious conduct that is within the scope of employment. Although respondeat superior usually applies to make an employer liable for an employee’s negligence, it also can make an employer liable for an employee’s intentional tort. An intentional tort would be particularly likely to be within the scope of empLoyment if the employee in question is doing a type of work, such as being a bouncer or security guard, that involves some use of physical force. Here, the bouncer would be liable to the customer for battery, and the owner of the bar would be vicariously liable for the battery. Choice (A) is incorrect because the fact that fights were reasonably foreseeable would be very significant for a negligence claim, but not crucial for an intentional tort claim. Choice (C) is incorrect because the owner could be liable for battery through respondeat superior even though he did not personally touch the customer. Choice (D) is incorrect because, as stated above, the owner would be liable for battery through respondeat superior.
- One Sunday afternoon, a sports fan went to a professional football game. While standing in line at a concession stand, he got into an argument with three men ahead of him in the line. The three men were friends attending the game together. One of the men punched the sports fan in the mouth. A fight ensued. During the fracas, one of the other three men hit the sports fan over the head with a bottle, which caused him to suffer a serious concussion. The sports fan does not know for certain which of the three men struck him with the bottle.
If the sports fan asserts a claim against the man who punched him in the mouth to recover damages for the head injury, will the sports fan prevail?
(A) Yes, because the man who punched him in the mouth was the instigator who struck the first blow that started the fracas.
(B) Yes, because the man who punched him in the mouth and the other men were acting in concert.
(C) No, because the man who punched him in the mouth did not actually strike the sports fan with the bottle.
(D) No, because the sports fan cannot offer proof as to which of the men struck the sports fan over the head with the bottle.
- (B) Whenever two or more persons commit tortious acts in concert, each becomes subject to Liability for the acts of the others, as well as for his own acts. Since the three men who fought with the sports fan were acting in concert, each would be liable for the head injury suffered by the sports fan, regardless of which of the three actually hit the sports fan’s head with the bottle. Choice (A) is incorrect because it does not matter that the man who punched him in the mouth was the instigator, unless he and the other two men were acting in concert. Choices (C) and (D) are incorrect because whenever two or more persons commit tortious acts in concert, each becomes subject to liability for the acts of the others as well as his own act. Since the man who punched him in the mouth and the other two men were acting in concert, the man who punched him in the mouth would be liable even though he himself did not strike the sports fan with the bottle.
- A baseball fan purchased two tickets for a World Series baseball game. The fan contacted his best friend and invited him to go to the game. The friend, who was a fanatic baseball fan, eagerly agreed. The fan told the friend that the game started at 7:00 p.m. and that he would pick him up at about 5:00 p.m. so they could get there early to watch batting practice.
They were driving to the game together when the fan sped up to cross an intersection while the traffic signal was changing from amber to red. As he reached the intersection, the fan was traveling at 50 m.p.h. although the posted speed limit was 25 m.p.h. Simultaneously, a car entered the intersection on red and collided with the fan’s vehicle. The friend suffered a broken pelvis in the collision. This jurisdiction has adopted the following “modified” comparative negligence statute:
“A negligent plaintiff is entitled to obtain a recovery provided plaintiff’s negligence is not equal to or greater than that of the defendant’s; otherwise no recovery is permitted.”
Suppose the friend brings suit against the driver of the car that entered the intersection on the red light to recover damages for his injury. Ajury returned a special verdict with the following findings: (1) The fan was 55 percent negligent in speeding; (2) The driver was 45 percent negligent in driving through the red light; and (3) The friend suffered $100,000 in damages. As a result, the court should enter a judgment for the friend in the amount of
a. $100,000.
b. $55,000.
c. $45,000.
d. nothing, because the fan was more negligent
than the driver.
- (A) On all standardized examinations (whether it be the MBE, SAT, LSAT, or GRE), the examiner’s main goal is to hide the correct answer. The examiners try to accomplish this by employing “distracters” and “red herrings.” Here, the comparative negligence statute is the distracter. A comparative negligence statute only applies when there is a negligent plaintiff and one or more negligent defendants. Here, there were two negligent parties (the fan and the driver), but neither of them is the plaintiff. The plaintiff (the friend) was just a passenger who was riding in one of the cars and did nothing negligent. The comparative negligence statute, therefore, is irrelevant. The two negligent parties (the fan and the driver) are joint tortfeasors who can be held jointly and severally liable for the full amount of the plaintiff’s damages. Choice (A) is correct because the friend may recover 100 percent of the damages from the driver, who could then seek contribution from the fan. Choices (B), (C), and (D) are incorrect because the fan and the driver are joint tortfeasors who would be held jointly and severally liable for the full amount of the friend’s damages.
“Modified” v. “Pure” Comparative Negligence
“Pure” comparative negLigence is simple. It means that where a jury assigns some fault to the plaintiff and some fault to the defendants, the plaintiff can recover for all the fault assigned to the defendants, no matter how high or low it may be compared to the plaintiff’s own fault. For example, even if the plaintiff was 99 percent at fault and the defendant was only 1 percent at fault, the plaintiff can still recover 1 percent of the damages for the plaintiffs injury.
In jurisdictions with “modified” comparative negligence rules, there is a cutoff point at which the plaintiffs share of the fault becomes so high that the plaintiffs claim is barred, and the plaintiff will recover nothing. In some jurisdictions, the plaintiff’s recovery is barred if the plaintiffs share of the fault is greater than the defendant’s share (e.g., if the plaintiff was 50 percent at fault and the defendant was 50 percent at fault, the plaintiff could recover 50 percent of the damages, but if the plaintiff was 51 percent at fault and the defendant was 49 percent at fault, the plaintiff would recover nothing). In other jurisdictions, the pLaintiffs recovery is barred if the plaintiffs share of the fault is equal to or greater than the defendant’s share (e.g., if the plaintiff was 49 percent at fault and the defendant was 51 percent at fault, the plaintiff could recover 51 percent of the damages, but if the plaintiff was 50 percent at fault and the defendant was 50 percent at fault, the plaintiff would recover nothing). Moreover, jurisdictions that have modified comparative fault rules also differ on how the comparisons are made if there are multiple negligent defendants. In some states, the plaintiffs share of the fault is compared separately with that of each individual defendant. Under the “unit rule” used in some other jurisdictions, the plaintiffs share of the fault is compared with the sum of all the defendants’ shares.
Example: Plaintiff suffers $100,000 damages and is 40 percent at fault. Defendant 1 is 10 percent at fault and Defendant 2 is 50 percent at fault. Analysis: Plaintiff can recover $60,000 in a pure comparative negligence jurisdiction. In a modified comparative negligence jurisdiction in which the plaintiffs share is compared separately to each defendant’s share, the plaintiff would be able to recover nothing from Defendant 1 (because the plaintiffs 40 percent share exceeds the 10 percent share of Defendant 1), but would be able to recover $50,000 from Defendant 2 (i.e., 50 percent of the $100,000). By contrast, under the unit rule, the plaintiffs share of the fault would be compared to the sum of all the defendants’ shares. Under that rule, the plaintiff could recover $60,000 from the defendants because the plaintiffs negligence would be less than the aggregate negligence of both defendants. Exam Tip: On questions involving multiple defendants, the facts will indicate whether to apply the unit rule in modified comparative negligence cases.
- At approximately 3:00 p.m. on December 3, a motorist was speeding down a street at the same time that a pedestrian was crossing the street. When the pedestrian started to cross the street, she saw the traffic light was red. She momentarily stopped, thought about waiting until the light turned green, but then decided to cross anyway. As the pedestrian was halfway across the street, the motorist, who was driving 50 m.p.h. in a 25 m.p.h. zone, struck her with his vehicle. The pedestrian suffered a broken pelvis and internal injuries in the accident. While in the hospital, the pedestrian’s insurance company paid $10,000 of her medical expenses. Six months later, the pedestrian brought suit against the motorist to recover damages for all her medical expenses. This jurisdiction has a “modified” comparative negligence statute in effect.
Suppose a jury returned a special verdict with the following findings: (1) The motorist was 60 percent negligent in speeding; (2) The pedestrian was 40 percent at fault in crossing the street against the red light; and (3) The pedestrian suffered damages from the accident totaling $50,000. After the verdict, the court was advised that the pedestrian’s insurance company had already paid the pedestrian $10,000. As a result, the court should enter a judgment for the pedestrian in the amount of
(A) $50,000, because the motorist’s negligence was greater than the pedestrian’s.
(B) $30,000, the proportion of the pedestrian’s damages caused by the motorist’s negligence, but the payment of $10,000 in hospital expenses will be disregarded under the collateral source rule.
(C) $20,000, the proportion of the pedestrian’s damages caused by her own negligence, but the payment of $10,000 in hospital expenses will be disregarded under the collateral source rule.
(D) $10,000, the proportion of the pedestrian’s damages caused by her own negligence, less the $10,000 in hospital expenses already paid by the insurance company.
- (B) Students must be cognizant of the distinction between “modified” comparative negligence and “pure” comparative negligence. In a modified comparative negligence jurisdiction, there will be a cutoff point at which the plaintiffs share of the fault becomes so great that the plaintiffs recovery is barred completely. In some jurisdictions, that cutoff point will be when the plaintiffs share of the fault is greater than that of the defendant’s. In other jurisdictions, it will be when the plaintiffs share is equal to or greater than that of the defendant’s. In the present question, the pedestrian’s negligence was less than that of the motorist’s negLigence, so the pedestrian will be entitled to recover in a modified comparative negligence jurisdiction. Choice (B) is correct because her recovery will be $30,000 (i.e., the total amount of damages of $50,000 reduced by 40 percent, which is the percentage of fault attributable to the plaintifl. Under the “collateral source” rule, a plaintiffs recovery is not reduced by virtue of the plaintiff having received compensation from sources other than the defendant. For example, if people feel sorry for the injured plaintiff and give her gifts, those gifts do not reduce the amount that the defendant owes for causing the plaintiffs injury. Likewise, the $10,000 paid by the pedestrian’s insurance company does not reduce her recovery from the motorist. Choice (A) is incorrect because under comparative negligence, the plaintiff recovers only for the portion of the fault assigned to the defendant. Choice (C) is incorrect because the amount that the pedestrian can recover is the proportion of the damages caused by the motorist’s negligence ($30,000), not the pedestrian’s own negligence ($20,000). Choice (D) is incorrect for the same reason that choice (C) is incorrect, and also because the damages will not be reduced by the $10,000 payment made by the pedestrian’s insurance company.
- A buyer purchased a new van from a car dealer. Two weeks later, the buyer was driving to work when the brakes suddenly failed. The buyer tried to stop the van for a red light, but the brakes failed to operate. As a consequence, the buyer drove through the red light and collided with a car driven by a woman.
Subsequently, the woman asserted a claim against the buyer to recover for the injuries she suffered in the accident. At trial, the only evidence offered by the plaintiff concerning the cause of the accident was the testimony of an engineering expert. He testified that a manufacturing defect had caused the brakes to suddenly fail.
Based on the facts stated above, a motion by the buyer to dismiss at the end of the woman’s case should be
(A) granted, because the woman presented no evidence that the buyer was negligent.
(B) granted, because the woman was neither the user nor the consumer of the defective product.
(C) denied, because the buyer had a non-delegable duty to maintain the brakes in a safe condition.
(D) denied, because the buyer is strictly liable for injuries caused by a manufacturing defect in an auto that he had purchased.
- (A) This question deals with burden of proof and requires a two-step analysis. First, it is necessary to recognize that the woman is bringing a negligence action against the buyer. Certainly, the woman cannot be suing the buyer for strict liability because the buyer is not a manufacturer or seller of the automobile. Next, on the issue of negligence,’ the question becomes the sufficiency of the evidence. In civil suits, unlike criminal prosecutions, the burden of proof does not require that the jury be convinced beyond all reasonable doubt, but only that they be persuaded that a preponderance of the evidence is in favor of the party sustaining the burden. Thus, the burden of pro.of of the defendant’s negligence is on the plaintiff to show the defendant’s negligence by a preponderance of the evidence. Because the woman presented no evidence that the buyer was negligent, the court should sustain the buyer’s motion to dismiss. Choice (B) is incorrect because it does not matter whether the woman was a user or consumer of the product. The woman’s claim fails because there is no proof that the buyer was negligent, not because the woman was not a user or consumer of the van. Choice (C) is incorrect because the issue here is not whether a duty existed. The buyer undoubtedly had a duty to exercise reasonable care, in maintaining the van as well as in driving. Although a duty existed, the issue here is whether there was sufficient evidence that the buyer breached the duty, and there is no such evidence. Choice (D) is incorrect because strict liability in tort would apply to a person engaged in the business of selling the defective product, and the buyer was a purchaser, rather than a seller of the van.
- A laboratory manufactures nitroglycerin (a heavy, oily, explosive, poisonous liquid used chiefly in making dynamite) at its main plant. An artist who specialized in making delicate glass sculptures had a studio two blocks away from the laboratory’s plant. Late one evening, there was an explosion at the laboratory’s plant. The force of the explosion caused the artist’s studio to be shaken, which resulted in the destruction of valuable artwork in the studio.
The artist now asserts a tort action against the laboratory to recover damages. Which of the following, if established, would furnish the laboratory with its best possible defense?
(A) The laboratory used extraordinary care in the manufacture and storage of nitroglycerin and was not guilty of any negligence that was causally connected with the explosion.
(B) The laboratory has a contract with the federal government whereby all the nitroglycerin manufactured at its plant is used in U.S. military weapons systems.
(C) The explosion was caused when lightning (an act of God) struck the plant during an electrical storm.
(D) The harm that the artist suffered would not have resulted but for the abnormal fragility of the artist’s work.
- (D) Strict liability may be imposed on one who carries on an abnormally dangerous or ultrahazardous activity. The person will be liable for harm resulting from the activity even if he exercised the utmost care to prevent the harm. The plaintiff must prove, however, that the defendant’s abnormally dangerous activity was both an actual and proximate cause of the plaintiff’s harm. This means the defendant will be strictly liable only if the harm results from the risk that makes the defendant’s activity abnormally dangerous. Strict liability will not be imposed if the harm occurred only because of the abnormal sensitivities of the plaintiff’s activities. Choice (A) is incorrect because the fact that a defendant was very careful will not prevent him or her from behind held strictly liable for an abnormally dangerous activity. Choice (B) is incorrect because even though most courts have held that governments are not subject to strict liability for abnormally dangerous activities, most courts have heLd that this immunity does not carry over to the contractors performing work for the government. Choice (C) is incorrect because strict liability for an abnormally dangerous activity can be imposed even if the harm triggered the operation of an unexpected force of nature. Even unexpected lightning, therefore, would not relieve the laboratory of strict liability for the explosion.
- A woman was seven months pregnant with her first child. One afternoon, the woman was in the kitchen of her home preparing a snack when she glanced out the window and saw a horrible crime being committed. The woman watched in horror as a local gang member pulled out a handgun and used it to shoot the victim, a member of a rival gang. The gang member did not know anyone was witnessing his act of shooting the victim. The woman watched as the victim collapsed and fell dead on her neighbor’s front lawn. As a result of her shock from viewing this horrible incident, the woman suffered a miscarriage.
If the woman asserts a claim against the local gang member who shot the victim, the woman will
(A) recover, because she suffered a physical injury, rather than just emotional harm.
(B) recover, because the gang member acted intentionally in shooting the victim.
(C) not recover, because the woman was not within the zone of danger.
(D) not recover, because the gang member was unaware that the woman was watching.
- (D) The issue presented is whether a plaintiff may recover for intentional infliction of emotional distress where the target of the defendant’s extreme and outrageous act is some other person, not the plaintiff. A plaintiff can recover in either of the following two situations: (1) The plaintiff is present when the defendant does something extreme and outrageous to a third person, the defendant is aware of the plaintiff’s presence, and the plaintiff is a close family relative of the third person who is the target of the defendant’s extreme and outrageous act. (2) The plaintiff is present when the defendant does something extreme and outrageous to a third person, the defendant is aware of the plaintiff’s presence, and the shock of witnessing the incident is so horrible that it causes the plaintiff to suffer bodily injury (such as a heart attack, stroke, etc.) rather than just emotional distress. Here, the woman cannot recover against the gang member for intentional infliction of emotional distress under either of these two rules, because the gang member was unaware of the woman’s presence. Choice (A) is incorrect because the fact that the woman suffered a physical injury is not sufficient to produce liability unless the defendant was aware of the woman’s presence. Choice (B) is incorrect because the fact that the gang member acted intentionally is not sufficient to produce liability unless he was aware of the woman’s presence. Choice (C) is incorrect because zone of danger is only in issue when a plaintiff is claiming emotional distress suffered for what almost happened to them. For example, if a plaintiff claims that she suffered emotionaL distress at aLmost being hit by a speeding car, the plaintiff would have to show that she was within the zone of danger of being hit by the car. Therefore, zone of danger would not apply to this fact situation.
- Three friends decided to go bowling together. They made a friendly wager on the match, with the winner receiving a free beer from the other two guys. In the second frame of the first game, the first friend apparently got a strike when he knocked down all 10 pins. However, the second friend accused the first friend of fouling because his foot went over the line. The first friend denied fouling and wanted to mark down a strike on the scorecard. The second friend refused to give the first friend a strike and wrote in a zero on the scoring sheet. The first friend became enraged.
The second friend then went to bowl his frame. As the second friend turned his back, the first friend approached from behind with a bowling ball in his hand. The first friend then raised the bowling ball threatening to hit the back of the second friend’s head. The second friend, who had his back turned, did not see the first friend’s actions. The third friend saw what the first friend was about to do and could easily have warned the second friend; but the third friend remained silent. The first friend then struck the second friend on the back of the head with the bowling ball, causing a deep scalp wound.
If the second friend asserts a cause of action against the third friend, he should
(A) prevail, because the third friend was aware of the danger.
(B) prevail, on account of the third friend’s omission to act.
(C) not prevail, because the first friend was responsible for his injury.
(D) not prevail, because the third friend was under no duty to warn the second friend of the danger.
- (D) The fact that the defendant reaLizes or should realize that action on his part is necessary for another’s aid or protection does not by itself impose a duty to take such action. There are situations where a special relationship between the parties (e.g., common carrier—passengers and innkeeper—guests) wiLl impose a duty to act. However, in this question there is no special relationship between the parties. Despite the fact that the third friend was aware of the danger, he did not have a duty to warn the second friend. Choice (D) is thus correct. Choice (A) is incorrect because although the third friend was aware of the danger, he did not have a duty to warn the second friend. Choice (B) is incorrect because the defendant cannot be held liable for an omission unless the defendant had a duty to act. Choice (C) is incorrect because there are situations in which more than one person can be held liable and responsible for an injury to a plaintiff. Here, the reason why the third friend is not liable is not because the first friend is responsible but, instead, because the third friend was under no legal duty to act.
- During a baseball game, a spectator watching the game started taunting one of the players for striking out. Annoyed by the taunts, the player started an argument with the umpire. When the umpire told the player to be quiet and go back to his team’s bench, the player became enraged and savagely attacked the umpire, hitting him in the head with a bat. The spectator who had taunted the player was overcome by humiliation and horror at what occurred, because he was a good friend of the umpire. The spectator suffered no physical injury, but experienced severe nervous shock and brooded over the incident for months.
If the spectator asserts an action for intentional infliction of emotional distress against the baseball player, the spectator will
(A) win, because the player’s conduct was extreme and outrageous.
(B) win, because he was present and witnessed the attack.
(C) lose, because he was partly responsible for causing the attack to occur.
(D) lose, because the umpire was not an immediate family member of the spectator.
- (D) A plaintiff generally cannot recover for intentional infliction of emotional distress unless the plaintiff is the target of the defendant’s extreme and outrageous act. However, a plaintiff can recover for emotionaL distress caused by seeing the defendant do something extreme and outrageous to another person if (1) the plaintiff is present when the defendant does the extreme and outrageous thing to the person, the defendant is aware of the plaintiffs presence, and the plaintiff is a close family relative of the person who is the target of the defendant’s extreme and outrageous act; (2) the plaintiff is present when the defendant does something extreme and outrageous to a third person, the defendant is aware of the plaintiffs presence, and the shock of witnessing the incident is so horrible that it causes the plaintiff to suffer bodily injury (such as a heart attack, stroke, etc.), rather than just emotional distress. Here, the spectator did not suffer any physical injury, so he can recover for intentional infliction of emotional distress only if he is related to the umpire, which, under the facts, he is not. Therefore, Choice (D) is the best answer. Choice (A) is incorrect because even though the player’s conduct was extreme and outrageous, the spectator cannot recover unless he was a family member of the umpire. Choice (B) is incorrect because mere presence is not by itself enough to permit a plaintiff to recover for intentional infliction of emotional distress where the plaintiff was not the target of the defendant’s extreme and outrageous conduct. Choice (C) is incorrect because the spectator’s taunting of the baseball player may have been impolite, but there is no legal rule under which it would bar recovery under tort law.
- While on vacation, two friends went scuba diving in the ocean. Each of them was equipped with a spear gun, which they planned to use to kill tuna. As they were swimming, both men saw what appeared to be a large fin entangled in a nearby coral reef. Simultaneously, they shot their spear guns at the projecting wing-like figure. Unbeknownst to either of the friends, it was not a fish but rather another scuba diver. The victim was struck and slightly wounded by one of the spears.
The victim asserts a claim for damages against the two friends. At trial, the judge should instruct the jury to rule
(A) in favor of both defendants if no evidence is presented showing who actually shot the spear that injured the victim.
(B) against both defendants jointly unless one of them proves that he did not shoot the spear that struck the victim.
(C) against each defendant for one-half of the amount of damages, because they both acted independently and not jointly.
(D) against each defendant for one-half of the amount of damages, because they both shot their spear guns simultaneously.
- (B) A plaintiff is generally required to identify the person who was the cause of the plaintiffs injury. However, the doctrine of “alternative liability” or “alternative cause liability” can assist a plaintiff unable to prove which one out of a group of defendants caused the plaintiffs injury. This doctrine applies where the plaintiff sues a group of defendants and shows that every one of the defendants engaged in the same tortious conduct, one of the defendants caused the plaintiffs injury, and the plaintiff is unable to determine which one was the cause. The burden of proof on the issue of who actually caused the harm will be shifted to the defendants. If a defendant can prove that it was not the cause, that defendant will avoid liability. However, defendants who cannot exculpate themselves (i.e., cannot prove they were not the cause) will be held jointly and severally liable for the plaintiffs injury. This is the rule enunciated in the case of Summers v. rice, 199 R2d 1 (Cal. 1948), where two hunters negligently fired shotguns in a plaintiffs direction. One of the hunters hit the plaintiff in the eye, but the plaintiff could not identify which hunter fired the shot that hit his eye. The plaintiff sued both hunters, and the court held that the hunters would be jointly and severally liable unless they could prove who actually caused the plaintiffs injury. Choice (B) is, therefore, correct. Choice (A) is incorrect because it overlooks the alternative liability rule under which the plaintiff may be able to prevail. Choices (C) and (D) are incorrect because if the doctrine of alternative liability is used successfully here, the defendants each will be jointly and severally liable for the plaintiff’s entire injury, rather than each being liable for half the injury.
- A driver and passenger were driving to work in the former’s automobile one morning. As they were traveling at a speed of 20 m.p.h. (which was within the posted speed limit), the passenger suddenly pointed to an overturned vehicle along the side of the highway and said, “Look at that car upside down.” The driver turned to look at the overturned vehicle. As he was looking toward the side of the road, the driver failed to see an abandoned vehicle with a flat tire in the highway about 200 feet in front of his approaching auto. Seconds later, the driver crashed into the rear of the abandoned auto and was injured. The jurisdiction has a relevant comparative negligence statute in effect.
If the driver asserts a claim against the owner of the abandoned auto, the most likely result is that the driver will
(A) recover all of his damages, because the defendant created a dangerous condition.
(B) recover only a portion of damages, because the abandoned auto was in plain view.
(C) recover nothing, because he had the last clear chance to avoid the collision.
(D) recover nothing, because the passenger’s act was a supervening cause.
- (B) As a general rule, comparative negligence statutes have the effect of apportioning damages based on the parties’ respective degrees of fault. For example, if the defendant’s fault is found to be twice as great as that of the plaintiff, the latter will recover two-thirds of his damages and bear the remainder of his loss himself. The driver was negligent in failing to keep a proper lookout while driving. By the same token, the defendant was also at fault because he left an abandoned auto in the middle of the highway. As a consequence, the plaintiff’s recovery will be diminished in proportion to his negligence. Choice (A) is incorrect because the plaintiff’s recovery will be diminished in proportion to his own negligence. Therefore, the plaintiff will not recover all of his damages. Choice (C) is incorrect because the “last clear chance” doctrine is applied in contributory negligence, not comparative negligence jurisdictions. Choice (D) is incorrect because the accident was a reasonably foreseeable result of the defendant’s negligence, even though the passenger’s action also played a role in bringing about the accident. There are often many events, forces, and actions that intervene between a defendant’s negligence and a plaintiff’s injury. Those intervening things generally are not supervening causes breaking the chain of proximate causation between the defendant’s negligence and the plaintiff’s injury. Although it is difficult to define exactly when an intervening cause will be treated as supervening and cut off the defendant’s liability, one of the most important factors is foreseeability. A foreseeable intervening act will rarely be treated as supervening. Here, the passenger’s action is within the scope of the foreseeable risk created by the defendant’s negligence, because the fact that passengers often briefly distract drivers is one of the reasons it is negLigent to abandon an automobile in the middle of the highway.
- A hardware store had a public pay telephone attached to the outside wall of its building. The telephone was owned and operated by the local telephone company.
On Thursday, November 25, the store was closed for Thanksgiving. A resident, who lived two doors away, walked to the hardware store to use the telephone. He wanted to call his son and daughter- in-law to wish them a happy Thanksgiving, but his home phone was out of order. The resident picked up the receiver, inserted coins for the cost of the call, and promptly received an electric shock. Although the resident was momentarily stunned, he did not suffer any pain or injuries. Unbeknownst to the resident, a technician employed by the telephone company had incorrectly rewired the telephone the previous day, causing a short circuit in the telephone mechanism.
If the resident institutes a personal injury action for negligence against the telephone company, he will most likely
(A) recover, because the technician’s error would constitute the proximate cause of the resident’s injuries.
(B) recover, because the technician’s error was a substantial factor in causing the resident’s injuries.
(C) recover, under the doctrine of respondeat superior
(D) not recover, because the resident did not suffer any injuries.
- (D) A negligence action requires proof that the plaintiff suffered some actual harm. Consequently, because the resident did not suffer any pain or other harm from the electrical shock, he would be unsuccessful in his negligence action. Choices (A), (B), and (C) are incorrect because in order for a plaintiff to prevail in a negligence cause of action, there must be some harm or injury to the plaintiff’s person or property.
- During a run one morning, a jogger passed a convenience store. He stopped and went into the store to use the bathroom available inside the store.
Which of the following would best describe the jogger’s legal status in his utilization of the bathroom?
(A) Licensee.
(B) Gratuitous licensee.
(C) Public invitee.
(D) Business visitor.
- (C) The jogger’s LegaL status is that of a public invitee. A public invitee is a person who is invited to enter or remain on land as a member of the public for a purpose for which the land is held open to the public. Choices (A) and (B) are incorrect because a person who enters land held open to the public is an invitee, not a mere licensee. If the jogger stopped at a friend’s house, for example, and received permission to enter and use the bathroom in the house, the jogger then would be a licensee. Choice (D) is incorrect because the jogger did not enter the Land in order to carry out any business dealings with the possessor or employees of the possessor.
- A man invited several friends to come over to his house to watch a movie on television. While they were watching the movie, freezing rain fell outside, coating everything with a thin layer of very slippery ice. When the movie ended, one of the man’s guests slipped on the ice as soon as he took a step out of the door and onto the front stoop of the man’s house.
Which of the following would best describe the duty of care owed by the man to his guest?
(A) No duty of care.
(B) A duty to inspect the premises for unknown dangers and disclose their existence to others.
(C) A duty to warn of any known dangerous condition on the premises.
(D) An absolute duty of care.
- (C) First, the guest is a licensee, who, in the broadest sense, includes anyone who comes upon the land with a privilege arising from the consent of the possessor. Next, what duty of care does a possessor owe to a Licensee? As to passive conditions on the Land, it is still the settled rule that the possessor is under no obligation to the licensee with respect to anything that the possessor does not know. The duty is not to maintain the land in a safe condition, but to exercise reasonable care so that the licensee is aware of the danger known to the possessor. Choice (A) is incorrect because the possessor of the land owes a duty to exercise reasonable care with respect to dangerous conditions of which the possessor has knowledge. Choice (B) is incorrect because with respect to a licensee, the possessor of land does not have a duty to inspect and find dangers not already known to the possessor. Choice (D) is incorrect because the possessor of land does not have an absolute duty to a licensee; the duty is merely for the possessor to exercise reasonable care so that the licensee is aware of the danger.
- An owner owned a two-story building, which he leased to a tenant. The tenant established a hardware store on the first floor and equipped the second floor as an apartment in which he lived with his wife and children. The two floors were connected by an outside wooden staircase with a handrail. The staircase was in a dilapidated condition at the time the tenant entered into the leasehold agreement. When the tenant took possession of the building, he notified the owner about the condition of the staircase and insisted that it be repaired. Although the owner orally promised the tenant that he would remedy the condition, he failed to do so.
Later that evening, the tenant and his wife were receiving some friends for a small dinner party. One of the guests arrived at 7:00 p.m. and climbed the stairs to the second floor apartment. When the guest was half way up the stairway, which had not been repaired, it collapsed, seriously injuring her.
In a negligence action initiated by the guest to recover for injuries suffered as a result of her fall, she will most likely
(A) recover against the tenant only, because as a general rule, a lessor of land is not liable to his lessee or to others on the land for physical harm caused by any dangerous condition that existed when the lessee took possession.
(B) recover against the owner only, because a lessor of land is subject to liability for physical harm caused to his lessee and others upon the land by a condition of disrepair existing before the lessee has taken possession.
(C) recover against the owner only, because under the public use exception, a lessor who leases land for a purpose that involves the admission of the public is subject to liability for physical harm caused to such persons by a hazardous condition existing when the lessee takes possession.
(D) recover against the tenant and the owner, because both the lessor and the lessee would be liable to others for their failure to remedy the defective staircase.
- (D) A lessee, as possessor of the property, has a duty to exercise reasonable care to warn licensees about dangers known to the lessee. A lessor of land, on the other hand, is generally not liable to his lessee or to others on the land for physical harm caused by any dangerous condition, whether natural or artificial, which existed when the lessee took possession. However, exceptions to this general rule incLude:
(1) situations where the Lessor contracts to repair; (2) undisclosed dangerous conditions known to the lessor; (3) land leased for purposes involving admission of the public; (4) parts of land retained in the lessor’s control, which lessee is entitled to use; or (5) where the lessor makes negligent repairs. Here, the tenant had a duty to exercise reasonable care to warn the guest about the known danger of the handrail of the staircase. Likewise, the owner had a duty because he agreed to repair the handrail. Choices (A), (B), and (C) are incorrect because the plaintiff was owed a duty by both the owner and the tenant to warn of the dangerous condition that existed on the property. This failure to warn resulted in the injury to the guest and, as such, she will have a cause of action against both the owner and the tenant.